You are on page 1of 94

Last Assignment for

PRC-04
(Introduction to Accounting)
December 2022 Attempt

Prepared by: Dawood Shahid CA & CIMA Affiliate, CPA, MPhil, MBA, OCE
1) Which of the following statements regarding inventory valuation is correct?
a) all inventory items should be b) the purchase price of items which
valued at cost have been held for the longest
period is an acceptable method
for valuing inventory
c) inventory valuation should d) all inventory items should be
exclude profit which has not yet valued at anticipated selling price
been earned less any cost which will be
incurred
2) In September 2020 ABC company wrote off an amount of Rs. 56,000 due from a
costumer who had become bankrupt however in January 2021 the company
unexpectedly received half of the amount due from the costumer:
How should the company account for this amount in its accounts for the year
ended on December 2021?
Select the most appropriate answer
a) as an accrual for Rs. 28,000 b) as increase in provision for
doubtful debt of Rs 28,000
c) as a prepayment of Rs 28,000 d) as other income of Rs 28,000
3) Which of the following statements are correct?
(I) sum of digit method and reducing balance method gives higher depreciation
in early years.
(II) straight line method and sum of digits method computes depreciation over
economic life.
a) Only 1 b) only 2
c) both are correct d) none is correct
4) Balance of which of the following groups of item would appear on opposite sides of
a trial balance?
a) inventory and drawings b) sales and returns outwards
c) carriage in and carriage out d) Trade receivables and returns
outwards
5) Drawing is a concept of:
a) Business entity b) Matching
c) Prudence d) Materiality
6) The matching concepts matches
a) assets and liabilities b) income with expenses
c) expenses with capital d) capital with reserves

Prepared by: Dawood Shahid CA & CIMA Affiliate, CPA, MPhil, MBA, OCE
7) In perpetual inventory system which of the following entries should be made to
recognize free samples given in charity?
a) debit inventory b) debit marketing expense
Credit cash Credit purchase
c) debit marketing expense d) debit purchases
Credit inventory Credit cash
8) Which of the following accounts should be debited to record credit sale of
inventory under perpetual system?
a) Only Accounts receivable b) Accounts receivable and
inventory
c) Accounts receivable, cost of d) Accounts receivable and cost
goods sold and inventory of goods sold
9) How would dishonor of customer’s cheque be recorded in cash book?
a) Recorded on receipt as well on b) not recorded in cash book
payment side
c) recorded on payment side only d) recorded on receipt side only
10) If net book value of an asset after charging first year depreciation is 9.2 million at
rate of 20% per annum. What will be depreciation of year 1 and year 2 under
reducing balance method?
a) Rs 1.41m and Rs 1.84m b) Rs 2.5m and Rs 1.84m
c) Rs 1.84m and Rs 1.84m d) Rs. 2.3m and Rs. 1.84m
11) Which one of the following statement is correct?
(A) All revenues are income but all incomes are not revenue
(B) Distribution of profit is also expense as it decreases the asset of business.
a) Only A b) only B
c) both statements are correct d) no statement is correct
12) BTL purchased goods for Rs. 1 million on credit from SPL on 5 th December. If BTL
pays the full amount within 10 days, it will be allowed a settlement discount of 5%.
BTL paid Rs. 950,000 on 12th December and availed the settlement discount. At
the time of payment, the accountant at BTL posted Rs. 950,000 on the payment
side of cash book. What will be the entry in the General Journal to record the
discount?
a) No entry b) Dr: SPL by Rs. 50,000
Cr: purchases Rs 50,000
c) Dr: Purchases by Rs. 50,000 d) Dr: SPL by Rs. 50,000
Cr: SPL Rs 50,000 Cr: other income Rs 50,000
13) Mr. A, the owner of the business, withdraws cash of Rs. 100,000. What is the
effect of this transaction?
a) Decrease in capital and b) Decrease in an asset and
decrease in an asset increase in another asset

Prepared by: Dawood Shahid CA & CIMA Affiliate, CPA, MPhil, MBA, OCE
c) Decrease in capital and increase d) Decrease in liability and decrease
in an liability in asset
14) Which of the following is correct?
(I) Purchase order is issued by seller to buyer.
(II) When an invoice is received from a supplier, it should be checked with goods
received note.
Select the most appropriate answer:
a) Only (I) is correct b) None if correct
c) Both are correct d) Only (II) is correct
15) Why is it necessary to account for accrued expenses?
Select the most appropriate answer:
a) So that current liabilities are not b) So that current assets are not
overstated. understated
c) So that profit is not understated d) So that profit is not overstated
16) Which of the following is correct?
(I) Asset is an economic resource controlled by an entity
(II) Equity is residual interest deducting current liabilities
Select the most appropriate answer:
a) Only (I) is correct b) None if correct
c) Both are correct d) Only (II) is correct
17) What is the relationship between useful life and scrape value?
a) Direct b) Inverse
c) Constant d) No relation
18) Which of the following reasons require an entry in the bank account in your
general ledger?
(I) A cheque from one of your customers amounting to Rs. 24,000 was returned
by the bank as unpaid, as the customer has gone bankrupt.
(II) A cheque amounting Rs. 18,000 in the name of your brother was erroneously
credited by the bank in your account.
Select the most appropriate answer:
a) Only (I) is correct b) None is correct
c) Both are correct d) Only (II) is correct
19) Which of the following statements are correct?
(I) The good debts do not require any special accounting treatment unlike bad
and doubtful debts.
(II) The doubtful debts must stay in the accounting records so that the business
continues to chase payment.
Select the most appropriate answer:

Prepared by: Dawood Shahid CA & CIMA Affiliate, CPA, MPhil, MBA, OCE
a) Only (I) is correct b) None is correct
c) Both are correct d) Only (II) is correct
20) Which of the following statements are correct?
(I) Credit entries decrease liabilities and increase income.
(II) Credit entries decrease expenses and increase assets.
Select the most appropriate answer:
a) Only (I) is correct b) None is correct
c) Both are correct d) Only (II) is correct
21) Write down of inventories to net realizable value should be recognised as:
Select the most appropriate answer:
a) current liabilities. b) An expense in the period in
which write-down occurs
c) Deferred expenses and d) An expense in the subsequent
transferred to profit and loss period in which such write down
account based on inventory is warranted.
movement to which write down
relates.
22) Which of the following statements are correct?
(I) Sales account is part of sales ledger
(II) Total of purchase return day book is posted to the debit side of return outward
account.
Select the most appropriate answer:
a) Only (I) is correct b) None is correct
c) Both are correct d) Only (II) is correct
23) Which of the following statements are correct?
(I) Sales account is part of sales ledger
(II) Total of sales day book is posted to the debit side of trade receivable account.
Select the most appropriate answer:
a) Only (I) is correct b) None is correct
c) Both are correct d) Only (II) is correct
24) The following information pertains to XYZ limited:
Rupees
Balance as per cash book (credit) 26,000
Balance as per bank statement (credit) 6,500
Unpresented cheques 45,500
The only reconciling item left is some cheques left banked and not yet credited in the
bank statements.
What is the total of cheques banked and not yet credited?

Prepared by: Dawood Shahid CA & CIMA Affiliate, CPA, MPhil, MBA, OCE
Select the most appropriate answer:
a) Rs 13,000 b) Rs 78,000
c) Rs. 65,000 d) Rs 26,000
25) Favorable balance means:
a) Credit balance in the cash book b) Debit balance in bank statement
c) Debit balance in the cash book d) All of these
26) Which two of the following statements are correct?
a) Bank reconciliation statement is b) Debit balance of cash book is a
prepared at the end of the favourable balance
accounting year only.
c) Bank reconciliation statement can d) To search causes of difference,
be started with cash book balance debit side of cash book (bank
only. column) is compared with the
credit column of Bank
statement and vice versa.
27) consider this graph and choose the correct
option:

a) The graph represent b) This graph represent book value


depreciation of asset under of asset under straight line
reducing balance method. method.
c) Both are correct d) None is correct
28) Which TWO of the following take interest in Financial statements of a Business?
a) Investors (shareholders) b)
c) d) Lenders
29) Which of the following statements are correct?
(I) In case of trade discount purchase price is not effected;
(II) In case of settlement discount, the sales is initially recognised at discounted
amount
a) Only (I) is correct b) None is correct
c) Both are correct d) Only (II) is correct
30) Which of the following statements are correct?
(I) If the cash discount is expected to be availed, then the purchases are
recognised at discounted amount.
(II) If the payment is expected to be made within the discounted period, then
sales are recognised at discounted amount

Prepared by: Dawood Shahid CA & CIMA Affiliate, CPA, MPhil, MBA, OCE
a) Only (I) is correct b) None is correct
c) Both are correct d) Only (II) is correct
31) Which adjusting entry will have same impact on accounting equation as
depreciation expense?
a) Unearned revenue. b) Accrued revenue
c) Expired prepaid rent d) Accrued expense.
32) A cheque is issued to withdraw cash for office use will be presented in cash book
at:
a) Payment side only b) Receipt side only
c) Payment and receipt both sides d) None of these
33) Which of the following entries will decrease owner’s equity?
a) Payment of expenses b) Payment of assets on credit.
Payment of assets Payment of liabilities
c) Repayment of loan d) Distribution by owners prepaid
Payment of liabilities expenses.
34) Which TWO of the following will have effect on assets and liabilities?
a) Purchase of asset on credit b) Payment of loan in cash
c) Credit sale of inventory d) Receipt from customers.
35) Which TWO of the following will have effect on assets and liabilities at the same
time?
a) Purchase of office furniture on b) Repayment of principal amount
credit of loan.
c) Credit sale of inventory d) Receipt of cash from customers.
36) What will define liability
a) A liability is something owed by b)
the business to someone else.
c) d)
37) What will define balance sheet?
a) Financial position of entity at b) Financial position of entity of
last day of financial year. complete year.
c) Financial statements of entity of d) Financial statement of entity at
complete year. last day of financial year.
38) Which of the following will capitalize the cost of vehicle?
a) Engine Repair b) Filling of petrol
c) Replacement of tyres d) Replacement of engine oil
39) Which of the following is correct?
(I) Fuel cost used in transportation business is direct cost.
(II) Sum of direct costs should exceed indirect costs.

Prepared by: Dawood Shahid CA & CIMA Affiliate, CPA, MPhil, MBA, OCE
a) Only I b) Only II
c) Both correct d) None correct
40) Which of the following is a business transaction?
(I) Bank set the limit of overdraft of Rs 10 million
(II) Interest charged by bank on overdraft
a) Only I is correct b) Only II correct
c) Both are correct d) None is correct
41) Which TWO of the following are business transactions?
a) A businessman purchased a vehicle for his private use, however he also
uses it for coming to office.
b) Furniture and fixtures laying in the office were destroyed in fire. Furniture
was owned by one of the partner and it was not in the use of business.
c) Electricity bill of one of firm’s partners was paid from business cash.
d) The proprietor provides a generator to office. It is presently not working
and it would have to be repaired before it can be used.
42) Write down of inventories to Net Realizable Value (NRV) should be recognised as:
a) Deferred expenses and transferred to profit and loss account based on
inventory movement to which write down relates.
b) An expense in the subsequent period in which such write down is warranted.
c) An expense in the period in which write down occurs.
d) Current liabilities
43) Which TWO of the following decrease the equity?
a) Write down to NRV b) Purchase lift truck on credit
c) Depreciation d) Borrowing loan on credit.
44) Which of the following is correct?
(I) The good debts do not require any special accounting treatment unlike bad
and doubtful debts.
(II) The doubtful debts must stay in the accounting records so that the business
continue to chase payment.
a) Only I b) Only II
c) Both d) None
45) Which of the following is correct?
(I) Cost of inventory is determined at lower of purchase price and Net Realizable
Value.
(II) Cost of inventory does not include indirect conversion cost.
a) Only I b) Only II
c) Both d) None

Prepared by: Dawood Shahid CA & CIMA Affiliate, CPA, MPhil, MBA, OCE
46) Asset may be overstated by removing (omitting) which TWO of the following
adjustments?
a) Expired insurance b) Depreciation
c) Unearned revenue d) Accrued salaries
47) A cheque received from supplier on return of faulty goods recorded in cash book?
a) Payment side of cash book b) Receipt side of cash book
c) Both payment side and receipt d) No entry
side.
48) Which of the following is/are correct?
(I) Bookkeeping is broader term than accounting
(II) Double entry bookkeeping is used to record dual aspect of transactions.
a) Only I b) Only II
c) Both d) None
49) Double entry accounting system can be applied in _______.
a) Cash basis accounting b) Accrual basis accounting
c) Both cash and accrual basis d) None of cash and accrual basis
accounting accounting.
50) Which of the following is/are correct?
(I) Accounting is broader term than Bookkeeping
(II) bookkeeping and accounting are two different but inter-connected functions.
a) Only I b) Only II
c) Both d) None
51) Which TWO of the following have no entry in periodic inventory system?
a) Normal loss b) Closing inventory
c) Cost of sales d) Abnormal loss
52) Which of the following is correct?
(I) Sales day book total balance is recorded in general Ledger
(II) Sales day book individual balance is recorded in sales ledger
a) Only I b) Only II
c) Both are correct d) Both incorrect.
53) Which of the following is correct?
(I) General journal is a book of prime entry for all transactions.
(II) Sales day book is a book of prime entry for credit notes issued to customers.
a) Only I b) Only II
c) Both are correct d) Both incorrect.
54) Which of the following is a business transaction?
(I) A person starts his business for Rs. 100,000 who has taken from his father.

Prepared by: Dawood Shahid CA & CIMA Affiliate, CPA, MPhil, MBA, OCE
(II) A purchase order is sent for delivery of goods within a week time
a) Only I b) Only II
c) Both are correct d) Both incorrect.
55) Which of the following is not a book of prime entry?
a) Sales day book b) Purchase return day book
c) General Journal d) General ledger
56) Which one is correct?
(I) Retail method in the retail industry for measuring inventories of large numbers
of rapidly changing items with similar margins for which it is impracticable to
use other costing methods.
(II) Specific identification method is necessary for inventory which is not
interchangeable.
a) Only I b) Only II
c) Both are correct d) Both incorrect.
57) Which of the following is correct in case of over-absorbed Overheads?
a) Actual OH > Absorbed OH b) Absorbed OH > Actual OH
c) Actual OH > under-Absorbed OH d) Absorbed OH > Production OH
58) 5 Used cars were sold by a Garments business and the amount has been received
in cash. The entry will be recoded in
a) Sales day book b) General Journal
c) Purchase Day book d) Cash book
59) Sales Account is found in
a) Sales day book b) Sales Ledger
c) General Ledger d) General Journal
60) Which of the following have effect on both assets and owners’ equity at the same
time?
a) Cash received by Credit b) Cash received by Cash sales
customers
c) Obtained a bank loan d) Owner introduced some cash
to the business
61) In which of the following case both transactions would decrease owner’s equity?
a) Purchase of an asset on credit. Payment of liabilities
b) Purchase of assets Payment of expenses
c) Distribution of assets to Payment of expenses
owner
d) Payment of liabilities Sale of goods
62) Another name of capital of:
a) Owner’s equity b) Net assets

Prepared by: Dawood Shahid CA & CIMA Affiliate, CPA, MPhil, MBA, OCE
c) Both a and b d) None of these
63) At the start of the year i.e. .1 October 2019, accrued rent was 120,000. The annual
rent was 720,000.
Payments have been made at following dates
1 October 2019 390,000
1 February 2019 300,000
1 March 2019 360,000
What will be the amount of prepaid or accrued rent at the end of the year i.e. 31
September 2020.
a) Prepaid rent Rs. 210,000 b) Accrued rent Rs. 210,000
c) Prepaid rent Rs. 360,000 d) None of these
64) Mr. Yousuf a tenant pays annual rent of Rs. 120,000. Payment is made quarterly
in advance on 1 January, 1 April, 1 July, 1 October. Which of the following should
be included in his accounts for the year ended 31 October 2021?
a) Rs. 20,000 prepayment. b) Rs 20,000 accrual
c) Rs 10,000 prepayment. d) Rs 10,000 accrual
65) On 1 January 2020, an entity rents out part of its premises to a tenant for Rs.
64,000 per month. As per agreement, rent is increased by 5% from start of each
year.
Payments totaling Rs. 736,000 were received from tenant during the year 2021
including rent for December 2020. Which figure should be reported in the entity’s
financial statements at 31 December 2021?
Income statement Current assets
a) Rs. 806,400 Rs 67,200
b) Rs. 803,200 Rs 128,000
c) Rs. 803,200 Rs 67,200
d) Rs. 806,400 Rs 134,400
66) An unearned revenue in the books of a receiver is likely to be
a) A prepaid expense in the books b) An unearned revenue in the
of the payer books of the payer
c) An accrued expense in the books d) An accrued revenue in the books
of the payer of the payer
67) Which of the following have same effect on the accounting equation as
Depreciation?
a) Unearned income b) Accrued expense
c) Prepaid expense d) Accrued income
68) Unearned income is treated as
a) Liability in Balance sheet b) Asset in (SOCI)
(SOFP).

Prepared by: Dawood Shahid CA & CIMA Affiliate, CPA, MPhil, MBA, OCE
c) Contra Asset d) Liability in SOCI.
69) For reconciling bank balances, unpresented cheques should be
a) Added to balance of bank b) Deducted from the balance of
statement and deducted from bank statement or added to
balance of cash book balance of cash book
c) Deducted from the balance of d) Added to balance of bank
bank statement and added to statement or deducted from
balance of cash book balance of cash book
70) A machine is purchased for 45 Million and has a residual value of 4.5 million and
useful life of 8 years. Calculate the rate of depreciation and amount of depreciation
for 1st year.
a) Rate= 25% and Depreciation is b) Rate= 12.5% and Depreciation
Rs. 5,062,500 is Rs. 5,062,500
c) Rate= 25% and Depreciation is d) Rate= 12.5% and Depreciation is
Rs. 5,620,500 Rs. 5,620,500
71) Bilal has started trading business. He deals in goods where prices are usually
rising over the time which of the following regarding selection of FIFO or Average
method in this case
a) FIFO will result in lower reported b) Average method will give more
profits accurate profit
c) Average method will lead higher d) Proft will be unaffected by the
cost of ending inventory selection of inventory method
72) Zameen, a property company, received cash totalling Rs. 838,600 from tenants
during the year ended 31 December 2016. Figures for rent in advance and in
arrears at the beginning and at the end of year were:
31 December 2015 31 December 2016
Rent receive in advance 102,600 88,700
Rent in Arrears (all subsequently received) 42,300 48,400
What amount should appear in company’s Statement of P&L for the year ended 31
December 2016 for rental income?
a) Rs. 818,600 b) Rs. 738,000
c) Rs. 939,200 d) Rs. 858,600
73) On year-end a shop owner has electricity bills of Rs. 27,000. During the year
electricity bills paid are Rs. 220,000. What adjustment will be required to utilize
expense account regarding the outstanding bills?
a) Rs. 27,000 Cr b) Rs. 220,000 Cr
c) Rs. 27,000 Dr d) None of these.
74) Which of the following statement is correct?
a) Debit balance as per bank b) Debit balance as per cash book
statement means a bank over means a bank overdraft

Prepared by: Dawood Shahid CA & CIMA Affiliate, CPA, MPhil, MBA, OCE
draft
c) Credit balance as per bank d) Credit balance as per cash book
statement means a bank over means an asset
draft
75) What is the impact on profit of the following?
Decrease in provision as compared to Bad debt expenses
last year
a) Profit will be decreased Profit will be increased
b) Profit will be increased Profit will be increased
c) Profit will be increased Profit will be decreased
d) Profit will be decreased Profit will be decreased
76) The correct form of accounting equation is
a) Assets + Liabilities = Equity b) Assets - Liabilities = Equity
c) Assets - Receivable = Equity d) Assets + Receivable = Equity
77) Advance payments are recognized as
a) Receivable b) payable
c) bad debt d) none of these
78) Long term assets without any physical existence but, possessing a value are
called
a) Intangible assets b) Fixed asset
c) Current assets d) Investments
79) Copyrights, Patents and Trademarks are examples of
a) Current assets b) Fixed assets
c) Intangible assets d) Investments
80) What from the following is NOT a current asset?
a) Patent rights b) Inventory
c) Cash d) Trade receivables
81) What from the following is/are NOT tangible asset(s)?
I. Patent rights
IL. Goodwill
III. Land
a) I only b) II only
c) I and II only d) I,II and III
82) Land, building and machinery always depreciated:
a) Correct b) Incorrect
83) Depreciable amount =
a) Cost of an asset + Residual value b) Cost of an asset - Residual value

Prepared by: Dawood Shahid CA & CIMA Affiliate, CPA, MPhil, MBA, OCE
c) Residual value - Cost of an asset d) None of these
84) What from the following is NOT a capital expense?
a) Purchase of property b) Purchase of office equipment
c) Replacement of a vehicle d) Repair of a vehicle
85) The expected disposal value of the asset (after deducting disposal costs) at the
end of its expected useful life is called
a) residual value b) net book value
c) depreciation d) substance over form
86) Depreciation is normally charged as
a) Payable b) Receivable
c) Expense d) advances
87) Raw materials that are remaining at the end of the reporting period are treated as
a) Liabilities b) expenses
c) fixed assets d) current assets
88) The goods that have been sold to the customers are treated as in the financial
statements.
a) Inventories b) expenses
c) income d) debt
89) The goods that have not been sold to the customers till the end of the reporting
period are considered as:
a) inventories b) expenses
c) sales d) purchases
90) A bank overdraft is shown as a/an _______ balance in the bank statement.
a) Debit b) credit
c) credit - debit d) none of these
91) Which from the following is included in the cost of purchases of an inventory item?
a) Abnormal loss b) Freight in
c) Rent of store d) Administrative Salaries
92) Cheques issued but not presented, cause the bank statement balance to be the
cash book balance.
a) greater than b) less than
c) equal to d) the two statements are irrelevant
93) The capital of a business is 100,000 and the liabilities are 40,000. What are the
total assets?
a) 40,000 b) 60,000
c) 100,000 d) 140,000

Prepared by: Dawood Shahid CA & CIMA Affiliate, CPA, MPhil, MBA, OCE
94) Any transaction which cannot be recorded in any other book of prime entry is
recorded in?
a) Cash book b) Petty cash book
c) General journal d) Day books
95) Nader traders (ND) keeps analyzed petty cash book using imprest system. The
imprest amount is 15,000 the following information is available for the month of
December 2021.
Date Description Rs.
Dec 1 Balance 15,000
Dec 5 Bought office stationery 2,500
Dec 7 Paid for office travel- Careem invoice 1,250
Dec 9 Paid for refilling of printer cartridges from market to 5,000
procurement staff
Dec 10 Received from procurement staff remaining balance - refilling 1,500
of printer cartridges
Dec 15 Paid for biscuits for owner’s guests 700
Dec 29 Purchased stamp papers from market 2,000
Petty cash balance as on 31 December 2021 before replenishment would be?
a) Rs 5,750 b) Rs 2,050
c) Rs 5,050 d) Rs 9,950
96) On the bank statement, cash deposited by the company is known as
a) Credit b) Debit
c) Liability d) Expenses
97) How we handle "Deposit in transit" in bank reconciliation?
a) Added to Bank Balance b) Subtracted From Bank Balance
c) Subtracted From the Cash Book d) Added to Cashbook Balance
Balance
98) Unpresented cheques also referred to as
a) Bounced cheques b) Outstanding cheques
c) Uncredited cheques d) Uncollected cheques
99) In cash book, bank charges of 50,000 was not recorded. what will be the correct
cash book adjustment?
a) it will be credited in cash book b) It will be debited in cash book
c) No adjustment needed in the cash d) Charges will be added to the
book cash book balance
100) In the books of account if a transaction is completely deleted, will it affect the
balancing of the trial balance?
a) No b) Yes
c) A transaction cannot be omitted d) None of these

Prepared by: Dawood Shahid CA & CIMA Affiliate, CPA, MPhil, MBA, OCE
101) When debit balance is equal to credit balance then the trial balance means
a) Account balances are correct b) Mathematically
Capital + Liabilities =Assets
c) No mistake in recording d) No mistake in posting entries
transactions to ledger accounts
102) Ledger is a
a) List of accounts b) Collection of accounts
c) Unique name of accounts d) None of these
103) Gross profit is
a) Cost of goods sold + Opening b) Sales - cost of goods sold
stock
c) Sales - Purchases d) Net profit – expenses
104) In order to find out the value of the closing stock during the end of the financial
year we,
a) do this by stocktaking b) deduct the cost of goods sold
from sales
c) deduct opening stock from the d) look in the stock account
cost of goods sold
105) Discounts received are
a) Buyer of goods granted b) Deducted when we receive cash
discount by seller
c) Given by us when we sell goods d) None of these
on credit
106) At the balance sheet date, the balance on the Accumulated Depreciation Account:
a) Simply deducted from the asset b) Transferred to Profit and Loss
in the Balance Sheet. Account
c) Transferred to the Asset Account d) Transferred to Depreciation
Account
107) If we take goods for own use, we should:
a) Debit Drawings Account, b) Debit Drawings Account:
Credit Purchases Account Credit Stock Account

c) Debit Sales Account: d) Debit Purchases Account:


Credit Stock Account Credit Drawings Account

108) A process of accounting that recognizes the impact of transactions on the financial
statements in the time periods when revenues and expenses occur instead of
when cash is received or disbursed is called _________ basis
a) Accrual b) Cash
c) a & b d) None

Prepared by: Dawood Shahid CA & CIMA Affiliate, CPA, MPhil, MBA, OCE
109) A process of accounting where revenue and expense recognition would occur
when cash is received and disbursed is called
a) Cash b) Accrual
c) a & b d) None
110) ____ is an increase in economic benefits during the accounting period in the form
of inflows or enhancements of assets or a decrease in liabilities, thereby
increasing equity and net worth.
a) Income b) Equity
c) Expenses d) None
111) The conventions, concepts, rules, and procedures that together make up accepted
accounting practice at any given time are called
a) AICPA b) GAAP
c) CAPA d) None of the above
112) What are Outstanding Expenses?
a) Expenses which are paid off in the b) The necessary purchases that keep
current balance sheet but not a business going from day-to-day.
incurred.
c) Type of expense that is due but d) None of the above
has not been paid.
113) _______ A/c is credited and_________ A/c is debited in case wages are paid for
construction of business premises
a) Cash, Wages b) Cash, Premises
c) Premises, Cash d) Wages, Cash
114) Among these statements which one is incorrect regarding journal entry?
a) The debited account titles are listed b) Journal entries show the effects of
first transactions
c) Each journal entry should begin with d) Journal entries provide account
a date balances
115) Who is an external user of financial statements?
a) Shareholders b) CEO
c) Manager d) Creditor
116) What is the common characteristic of all the assets owned by a company?
a) Intangible b) Long Life
c) Future Economic Benefits d) None of the above
117) What causes the decrease in Assets?
a) Cash Purchases b) creation of a Liabilities
c) Payment of Expenses d) Increase Retained Earnings

Prepared by: Dawood Shahid CA & CIMA Affiliate, CPA, MPhil, MBA, OCE
118) What cause an increases in the Assets & Equity?
a) Introduction of Fresh Capital b) Debts paid off
c) payment of expenses d) None of the above
119) What is a compound entry?
a) An entry involving more than two b) Accounting entry in which just one
accounts. account is debited and one is
credited.
c) An accounting entry in which there d) Entry that is used to record an
is more than one debit, more than interest transaction in the accounting
one credit, or more than one of records of a business.
both debits and credits.
120) The matching concept matches:
a) assets with liabilities b) income with expenses
c) capital with reserves d) expenses with capital
121) The purpose of depreciation is to
a) allocate the depreciable cost on a b) write the asset down to its realizable
systematic basis over the asset’s value each period.
useful life.
c) accumulate a fund for asset d) recognise that assets lose value
replacement. over time.
122) Zee bought goods on credit from Shan Traders (ST). The goods were not
according to the required specifications and therefore returned to ST. Which
document should Zee send to ST?
a) Credit note b) Debit note
c) Invoice d) Statement of account
123) International Financial Reporting Standards are issued by:
a) Financial Accounting Standards b) International Accounting
Board Standards Board.
c) International Accounting Standards d) Generally Accepted Accounting
Committee Principles Board
124) The cost of a wastebasket having an estimated useful life of 5 years is charged off
as an expense upon purchase. This is an example of the application of the:
a) consistency b) matching principle.
c) materiality concept d) historical cost principle
125) Calculate the value of closing stock:
Items Cost NRV
Product A 95,000 93,000
Product B 100,000 102,000
Product C 85,000 79,000
Product D 78,000 85,000

Prepared by: Dawood Shahid CA & CIMA Affiliate, CPA, MPhil, MBA, OCE
Product E 44,000 41,000

a) Rs 402,000 b) Rs 391,000
c) Rs 400,000 d) Rs 401,000
126) Accrual concept does not apply in which TWO of the following?
a) Understate assets b) Overstate assets
c) Understate liabilities d) Overstate liabilities
127) Which of the following is correct?
(I) An accrual is expense relating to next year but paid in current year.
(II) A prepaid is an expense relating to current year but not paid in current year.
a) Only I b) Only II
c) Both are correct d) Both incorrect.

128) If one asset is decreased, then which of the following might be increased?
a) Income b) Liability
c) Capital d) Expense
129) If one asset is decreased, then which of the following might be increased?
a) Asset b) Liability
c) Expense d) Both a and c
130) Bank transactions are not recorded in which of the following?
a) Single column cashbook b) Three column cashbook
c) Petty cashbook d) Double column cashbook
131) Debit always means
a) left side of an account b) right side of an account
c) increase d) increase in assets
132) Which of the following is NOT an asset that falls under the scope of IAS 16
Property, Plant and Equipment?
a) Tangible assets b) Assets held for the production or
supply of goods or services
c) Assets held for sale in the d) Assets expected to be used for
normal course of business more than one period
133) Which of the following statements is correct regarding trial balance?
a) Every credit balance represents b) Return inward has debit balance
income
c) Opening stock as well as closing d) Trial balance is a book of prime
stock appear in a trial balance entry
134) Which of the following statements is correct?
a) Directors of a company are b) A sole trader business is owned by

Prepared by: Dawood Shahid CA & CIMA Affiliate, CPA, MPhil, MBA, OCE
personally liable for any losses of the shareholders and operated by the
company proprietor
c) Partners are liable for losses in a d) A company is run by directors on
partnership equally regardless of their behalf of shareholders
profit sharing ratio
135) During the year 2019, an entity purchased a machine for Rs. 20 million to be used
for 6 years. Which of the following would represent residual value of this machine
in 2019?
a) Rs. 15 million can be currently b) Rs. 4 million can be currently
obtained from disposal of the obtained from disposal of a 6 year
machine in present condition old similar machine
c) Rs. 4 million can be currently d) Rs. 6 million can be obtained in
obtained from disposal of a 6 year old 2025 from disposal of 6 years old
similar machine in present condition similar machine
136) Allowance for doubtful debts’ account is:
a) liability account b) asset account
c) contra asset account d) expense account
137) Contra Asset account has what type of balance?
a) Debit b) Credit
c) Contra d) All of these
138) Which of the following does not require journal entry in periodic inventory method?
a) Abnormal loss b) Goods returned by a supplier
c) Closing inventory d) Normal loss
139) For reconciling bank balances, unpresented cheques should be:
a) added to balance of bank b) deducted from balance of bank
statement and deducted from statement or added to balance
balance of cash book of cash book
c) deducted from balance of bank d) added to balance of bank
statement and added to balance of statement or deducted from
cash book balance of cash book
140) Which of the following statement(s) is/are correct?
(I) Higher closing prepaid balance than previous year means that payment for the year
exceeds the expense for the year.
(II) Higher closing accrued income balance than previous year means that receipt for the
year exceeds the income for the year.
a) Only (I) is correct b) Only (II) is correct
c) Both are correct d) None is correct
141) If a property, plant and equipment remains idle for whole year, the depreciation
expense for the year will be NIL under:
a) straight line method b) reducing balance method

Prepared by: Dawood Shahid CA & CIMA Affiliate, CPA, MPhil, MBA, OCE
c) units of production method d) all of the above methods
142) if an asset (property, plant & equipment) is idle, then in which type of method no
depreciation is charged:
I) straight line method
II) units of production method
a) Only (I) is correct b) Only (II) is correct
c) Both are correct d) None is correct
143) On 4 August 2021 goods were sold on credit for Rs 125,000 with credit terms of
3/10 n/20. On 8 August 2021 goods of Rs 25,000 were returned by the buyer. How
much should the seller expect to receive if the buyer pays on 12 August 2021.
a) Rs 100,000 b) Rs 125,000
c) Rs 97,000 d) Rs 121,250
144) Which of the following statements about prudence is correct?
a) Assets and income must be b) The lowest possible values
understated while liabilities and should be applied to income and
expenses must be overstated. assets and the highest possible
values to expenses and liabilities.
c) Assets and income are not d) Assets and income are not
overstated while liabilities and overstated while liabilities and
expenses are not understated. expenses are not understated.
145) Income arises from increase in assets or decrease in a liability resulting in:
a) Decrease in equity other than b) Increase in equity other than
contribution from owner. contribution from owner.
c) Increase in equity including d) Decrease in equity including
contribution from owner. contribution from owner.
146) Net assets (Assets – Liabilities) of a business would be changed as a result of:
a) Non-current assets being b) A supplier balance being paid in
purchased on cash cheque
c) Wages being paid in cash d) Non-current assets being
purchased on credit
147) Which TWO of the following would NOT be recorded in General journal?
a) Increase in provision of doubtful b) Advance received from customer
debts
c) Write off of a trade receivable d) Receipt of a written off balance
balance
148) Which of the following document would require a double entry?
a) Sales order b) Sales invoices
c) Delivery note d) Statement of account

Prepared by: Dawood Shahid CA & CIMA Affiliate, CPA, MPhil, MBA, OCE
149) Which is the purpose of maintaining an allowance for doubtful debt receivables.
a) Records irrevocable debts b) Records the expense of
without taking them out of the irrevocable debts.
books of an entity.
c) Have an estimate of future d) Matches the estimate cost future
irrecoverable debts. irrecoverable debts against the
revenue earned in giving rise to
the potential irrecoverable debts.
150) A business has an overhead absorption rate of Rs 425 per machine hour based on
budgeted level of activity. Which is 12,400 hours. In the period covered by the
budget actual machine hours worked were 2% more than the budgeted hours and
actual overhead expense incurred was Rs. 5,638,900. What was the absorbed OH
for the period?
a) Rs. 5,638,000 b) Rs. 5,375,400
c) Rs. 5270000 d) Rs. 5,500,000
151) What entry should be recorded when actual OH expenditure is incurred
a) Cost of sale to Production OH b) Inventory WIP to production OH
c) Production OH to cash d) Production OH to cost of sale
152) Which of the following cost would most likely to be remain same at any level of
activity?
a) Indirect material cost b) Direct material cost
c) Factory rent d) Labour cost
153) Which TWO of the following items would be appearing in analysis of expenses by
nature?
a) Salaries exp b) Admin exp
c) Distribution exp d) Finance cost
154) A liability is classified as current liability if it is paid;
a) Out of current assets b) Within 1 year or normal
operating cycle whichever is
longer
c) Out of non-current assets d) Within 1 year or normal operating
cycle whichever is shorter
155) A liability is classified as current liability if it is paid;
a) It will be paid out of current assets b) It is payable within 1 year or
normal operating cycle
whichever is longer
c) If the entity has a right to defer d) Within 1 year or normal operating
settlement of the liability for atleast cycle whichever is shorter
12 months.

Prepared by: Dawood Shahid CA & CIMA Affiliate, CPA, MPhil, MBA, OCE
156) A company purchased an inventory item at purchase price at Rs 1800 and also
incurred freight in cost of Rs. 155. The item will be sold at a margin of 15% after
incurring freight out and packing cost of Rs 200 and Rs 150 respectively. At which
value the item should be carried in the books?
a) Rs 1,800 b) Rs 1,955
c) Rs 1,950 d) Rs 2,300
157) In preparing its financial statement for the current year an entity closing inventory
was understated by RS. 200,000. What will be the effect of this error if it remains
uncorrected?
a) The current year profit will be b) The current year profit will be
overstated and next year profit will understated and next year
be understated profit will be overstated
c) The current year profit will be d) The current year profit will be
overstated but there will be no understated but there will be no
effect on next year profit. effect on next year
158) Which group only contains indirect cost?
a) Carriage inwards , factory rent , b) Carriage inwards , raw materials ,
wages of factory supervisors wages of machine operators
c) Depreciation of machinery , d) Depreciation of machinery , raw
factory rent , wages of factory materials , wages of machine
supervisors operators
159) During the month of July 2020, a manufacturing business issued Rs. 20.2 million
of direct materials to the factory and Rs. 4.9 million of indirect materials. What is
the double entry for these issues of materials?
a) Dr. Materials Control Rs. 25.1m & Cr. b) Dr. WIP Rs. 4.9m & Dr. Production
WIP Rs. 4.9m & Cr. Production OH OH Rs. 20.2m & Cr. Materials
Rs. 20.2m Control Rs. 25.1m
c) Dr. WIP Rs. 20.2m & Dr. Production d) Dr. Materials Control Rs. 25.1m &
OH Rs. 4.9m & Cr. Materials Cr. WIP Rs. 20.2m & Cr. Production
Control Rs. 25.1m OH Rs. 4.9m
160) During month, the direct production workers worked for 810 hours including 190
hours of general overtime. The indirect workers worked for 230 hours, which
included 38 hours of general overtime. The direct production workers have a basis
rate of Rs. 790 per hour and the indirect have a basic rate of Rs. 590 per hour.
What is the amount that will be debited to the production overhead account
a) Rs. 639,000 b) Rs. 790,000
c) Rs. 775,600 d) Rs. 890,445

Prepared by: Dawood Shahid CA & CIMA Affiliate, CPA, MPhil, MBA, OCE
161) A entity engaged in manufacturing has policy of holding no inventory of any type
whether raw materials, WIP or finished goods. During an accounting period it only
produced goods from raw material purchased by it. Its cost of sales shall be equal
to:
a) Prime Cost+ Production b) Direct labour +Production
Overheads Overheads
c) Prime Cost d) Cost of direct materials consumed
162) Direct labor includes the wages of:
a) The shop foreman. b) an hourly worker producing the
product.
c) maintenance workers. d) administrators
163) Items of property, plant, and equipment should be recognized as assets when it is
probable that:
a) The future economic benefits b) The cost of the asset can be
associated with the asset will flow measured reliably.
to the enterprise
c) Asset is in good condition d) Both A and B
164) A company uses the reducing balance method to depreciate its fixed assets. the
annual rate is 20%. After three years the proportion of original cost still
underappreciated will be:
a) 51.2% b) 48.8%
c) 40.0% d) Impossible to determine. It depends
on the estimated residual value.
165) ___________Is the amount at which an asset is recorded in the balance sheet,
after deducting any accumulated depreciation and accumulated impairment
losses?
a) Carrying amount b) Cost
c) Depreciable amount d) Recoverable amount
166) An item can be converted easily into cash. In which section of the statement of
financial position would this item appear?
a) non-current assets b) current liabilities
c) current assets d) capital
167) Accrual basis of accounting records revenue when they are
a) Collected b) Earned
c) Contracted d) Readily available for use
168) Which of the following should not be shown as an intangible non-current asset?
a) Purchased goodwill b) Non-purchased goodwill
c) Development cost meeting the d) Other intangible assets having
criteria in IAS 38 readily ascertainable market

Prepared by: Dawood Shahid CA & CIMA Affiliate, CPA, MPhil, MBA, OCE
value
169) Statement of comprehensive income is prepared for
a) Last day of accounting year b) For whole year
170) Statement of financial position is prepared for
c) Last day of accounting year d) For whole year
171) The matching concept matches
a) assets with liabilities b) income with expenses
c) capital with reserves d) expenses with capital
172) An inexperienced accountant has prepared the current account of one of the
partners as follows:
Rs. Rs.
Interest on capital 33,000 Balance b/d 50,000
Balance c/d 40,000 Drawings 10,000
Share of residual profit 13,000
73,000 73,000
The balance brought down is entered correctly and the amounts of other entries
are correct. However, the accountant is not sure of the difference between debits
and credits.
What should be the correct amount of balance carried down?
a) Rs. 60,000 b) Rs. 70,000
c) Rs. 86,000 d) Rs. 96,000
173) Which of the following errors will create balance in a suspense account?
a) Repairs expense was considered b) Purchase of inventory was
as purchase of asset considered as purchase of non-
current asset

c) An invoice of Rs. 2,500 was totally d) Petty cash expenses of Rs. 500
omitted from the books were only credited to bank
account
174) The purpose of depreciation is to
a) allocate the depreciable cost on a b) write the asset down to its
systematic basis over the asset’s realisable value each period
useful life
c) accumulate a fund for asset d) recognise that assets lose value
replacement over time

Prepared by: Dawood Shahid CA & CIMA Affiliate, CPA, MPhil, MBA, OCE
175) Zee bought goods on credit from Shan Traders (ST). The goods were not
according to the required specifications and therefore returned to ST. Which
document should Zee send to ST?
a) Credit note b) Debit note
c) Invoice d) Statement of account
176) International Financial Reporting Standards are issued by:
a) Financial Accounting Standards b) International Accounting
Board Standards Board

c) International Accounting d) Generally Accepted Accounting


Standards Committee Principles Board

177) The cost of a wastebasket having an estimated useful life of 5 years is charged off
as an expense upon purchase. This is an example of the application of the:
a) consistency b) matching principle
c) materiality concept d) prudence concept.
178) Matching principal:
a) Determine whether an item is an asset or liability.
b) Addresses the relationship b/w the accounting database and the balance
sheet (SOFP).
c) Requires that the account of assets equals the amount of liabilities in the
balance sheet.
d) Determines that expenses related to revenue are to be reported in the
period when the revenue is recognised.
179) Debit always means
a) left side of an account b) right side of an account
c) increase d) increase in assets
180) how will these assets expire?
Prepaid rent Office supplies
(a) With the passage of time Through use and consumption
(b) Through use and consumption With the passage of time
(c) With the passage of time With the passage of time
(d) Through use and consumption Through use and consumption
181) An unearned revenue in the books of a receiver is likely to be
a) a prepaid expense in the books b) a prepaid expense in the books of
of the payer the payer

c) an accrued expense in the books d) an accrued revenue in the books


of the payer of the payer

Prepared by: Dawood Shahid CA & CIMA Affiliate, CPA, MPhil, MBA, OCE
182) Which of the following is NOT an asset that falls under the scope of IAS 16
Property, Plant and Equipment?
a) Tangible assets b) Assets held for the production or
supply of goods or services

c) Assets held for sale in the d) Assets expected to be used for


normal course of business more than one period

183) Which of the following statements is correct regarding trial balance?


a) Every credit balance represents b) Return inward has debit
income balance

c) Opening stock as well as closing d) Trial balance is a book of prime


stock appear in a trial balance entry

184) Which of the following statements is correct?


a) Directors of a company are b) A sole trader business is owned
personally liable for any losses of by shareholders and operated by
the company the proprietor

c) Partners are liable for losses in a d) A company is run by directors


partnership equally regardless of on behalf of shareholders
their profit-sharing ratio
185) Which of the following statement is correct?
(I) Directors are responsible for preparation of financial statements of a company
for public disclosure.
(II) Partners are required to prepare financial statements of a partnership for
public disclosure.
a) Only I b) Only II
c) Both are correct d) Both incorrect.
186) Interest charged on drawings of partners:
a) increases the profit available for b) decreases the profit available for
appropriation to partners appropriation to partners

c) increases expenses of the d) decreases expenses of partnership


partnership business business
187) A motor vehicle was purchased for Rs. 1,395,000 on 1 July 2018. It has an
estimated useful life of 5 years and a residual value of Rs. 255,000. If the sum of
the digits method of depreciation is used, what will be the carrying amount of
motor vehicle as at 30 June 2020?
a) Rs. 711,000 b) Rs. 558,000

Prepared by: Dawood Shahid CA & CIMA Affiliate, CPA, MPhil, MBA, OCE
c) Rs. 456,000 d) Rs. 303,000
188) A company purchased a delivery van for Rs. 1,700,000 with a salvage value of Rs.
260,000 on 1 October 2021. It has an estimated useful life of 4 years. Using the
straight line method of depreciation, how much depreciation expense should the
company recognise on 31 December 2021?
a) Rs.90,000 b) Rs. 360,000
c) Rs. 425,000 d) Rs. 106,250
189) During the year 2019, an entity purchased a machine for Rs. 20 million to be used
for 6 years. Which of the following would represent residual value of this machine
in 2019?
a) Rs. 15 million can be currently b) Rs. 4 million can be currently
obtained from disposal of the obtained from disposal of a 6
machine in present condition year old similar machine

c) Rs. 18 million can be obtained in d) Rs. 18 million can be obtained in


2025 from disposal of the machine 2025 from disposal of the
in present condition machine in present condition

190) A, B and C are partners in a firm sharing profit or loss in the ratio of 3:3:4. B is
entitled to an annual commission of 5% on sales. The sales and net profit earned
by the firm during the year are Rs. 12 million and Rs. 3 million respectively. What
is C's share of profit?
a) Rs. 720,000 b) Rs. 1,140,000
c) Rs. 1,560,000 d) Rs. 960,000
191) An entity received electricity bill for the month of June 2020. The bill will be paid on
10 July 2020. The entity's financial year ends on 30 June. How would the payment
be recorded?
a) Debit electricity expense and b) Debit utilities payable and credit
credit utilities payable electricity expense

c) Debit utilities payable and credit d) Debit electricity expense and credit
cash cash

192) Allowance for doubtful debts’ account is:


a) liability account b) asset account
c) contra asset account d) expense account
193) Which of the following does not require journal entry in periodic inventory method?
a) Abnormal loss b) Goods returned by a supplier
c) Closing inventory d) Normal loss

Prepared by: Dawood Shahid CA & CIMA Affiliate, CPA, MPhil, MBA, OCE
194) For reconciling bank balances, unpresented cheques should be:
a) added to balance of bank b) deducted from balance of bank
statement and deducted from statement or added to balance
balance of cash book of cash book
c) deducted from balance of bank d) added to balance of bank
statement and added to balance of statement or deducted from
cash book balance of cash book
195) Which of the following statement(s) is/are correct?
(I) Higher closing prepaid balance than previous year means that payment for the
year exceeds the expense for the year.
(II) Higher closing accrued income balance than previous year means that receipt
for the year exceeds the income for the year.
a) Only (I) is correct b) Only (II) is correct
c) Both are correct d) None is correct
196) Rent amounts totalling Rs. 150,000 were received during the year ended 31
December 2019. In this respect, opening and closing balances are as under:
31 Dec 2019 31 Dec 2018
----------- Rupees -----------
Unearned rent 16,000 10,000
Rent receivable 13,000 17,000
What should be the rent income for the year ended 31 December 2019?
a) Rs. 144,000 b) Rs. 152,000
c) Rs. 148,000 d) Rs. 140,000
197) A transaction not recorded in any other books of original entry is recorded in:
a) Sales day book b) Purchases day book
c) General journal d) Cash book
198) On 1 February 2019, a loan of Rs. 6,000,000 was taken from a bank for acquisition
of an office building. A portion of the building was rented out on 1 August 2019 at a
monthly rent of Rs. 45,000. Interest is payable at 15% per annum on 31 January
each year and rent is received half yearly in advance.
What amounts of interest payable and unearned rent should be shown in the
statement of financial position as on 31 December 2019?
a) Interest payable Rs. 900,000; b) Interest payable Rs. 825,000;
Unearned rent Rs. 270,000 Unearned rent Rs. 45,000

c) Interest payable Rs. 825,000; d) Interest payable Rs. 825.000;


Unearned rent Rs. 45,000 Unearned rent Rs. 250,000

Prepared by: Dawood Shahid CA & CIMA Affiliate, CPA, MPhil, MBA, OCE
199) An entity acquired laptops in exchange of desktops which have carrying amount of
Rs. 450,000 and fair value of Rs. 300,000 at the date of exchange. The list price of
the laptops acquired is Rs. 600,000. The entity is also required to pay cash of Rs.
275,000 in this exchange transaction.
The laptops should be initially recognized at:
a) Rs. 300,000 b) Rs. 575,000
c) Rs. 450,000 d) Rs. 600,000
200) Determine the balance as per bank statement using the following information:
Balance as per cash book: 49,100
Cheques received and deposited into the bank, but not yet credited in the 4,600
bank statement
Unpresented cheques; 6,300
Credit transfers appearing in the bank statement but not entered in the 3,400
cash book;

a) Rs. 52,500 b) Rs. 54,200


c) Rs. 58,800 d) Rs. 50,800
201) Which of the following statements is correct?
a) Debit balance as per bank b) Debit balance as per cash book
statement means a bank means a bank overdraft
overdraft
c) Credit balance as per bank d) Credit balance as per cash book
statement means a bank overdraft means an asset
202) If a property, plant and equipment remains idle for whole year, the depreciation
expense for the year will be NIL under:
a) straight line method b) reducing balance method
c) units of production method d) all of the above methods
203) On 1 July 2019 a business acquired a shop on rent which is payable quarterly in
advance. Payments of rent were made as follows:
Date Rupees
1 July 2019 75,000
30 September 2019 75,000
31 December 2019 75,000
31 March 2020 75,000
30 June 2020 75,000
What will be the rent expense charged to statement of profit or loss for the year ended
31 August 2020?
a) Rs. 375,000 b) Rs. 300,000
c) Rs. 75,000 d) Rs. 225,000

Prepared by: Dawood Shahid CA & CIMA Affiliate, CPA, MPhil, MBA, OCE
204) On 4 August 2021 goods were sold on credit for Rs 125,000 with credit terms of
3/10 n/20. On 8 August 2021 goods of Rs 25,000 were returned by the buyer. How
much should the seller expect to receive if the buyer pays on 12 August 2021.
a) Rs 100,000 b) Rs 125,000
c) Rs 97,000 d) Rs 121,250
205) On 30 June 2020, the balance in allowance for doubtful receivables showed Rs
16,000. During the year ended 30 June 2021, receivables of Rs 21,000 were
written off. If allowance for doubtful receivables at 30 June 2021 is Rs 18,000 then
what would be the total effect of bad and doubtful receivables on profit for the
years?
a) Decreased by Rs 19,000 b) Increased by Rs 19,000
c) Decreased by Rs 23,000 d) Increased by Rs 23,000
206) A liability is classified as current liability of it is paid:
a) Within one year or normal b) Within one year or normal
operating cycle, whichever is operating cycle, whichever is
longer shorter
c) Out of current assets d) Out of non-current assets
207) Karachi traders (KT) perform stock count on 10 January 2021 for the year ended
31 December 2021. The stock count revealed inventory value of Rs 480,000.
During 1 January to 10 January 2021, KT sold inventory for Rs 81,000 at a cost
plus mark-up of 35% and purchased inventory of Rs 38,000. The value of
inventory as at 31 December 2020 should be:
a) Rs 523,000 b) Rs 502,000
c) Rs 458,000 d) Rs 437,000
208) On 1 may 2019, an entity hired a firm at an annual fee of Rs 150,000 for
maintaining accounting records. On 1 may 2020 the contract was renewed for
further one year at a revised annual fee of Rs 210,000. The fee is paid quarterly in
arrears on 31 July, 31 October, 31 January and 30 April each year. What figure
should appear for fee in in the financial statements for the year ended 31
December 2020?
Statement of Financial Position Statement of Profit or loss
a) Prepaid of Rs 35,000 Expense of Rs 190,000
b) Prepaid of Rs 17,500 Expense of Rs 180,000
c) Accrual of Rs 35,000 Expense of Rs 190,000
d) Accrual of Rs 17,500 Expense of Rs 180,000
209) Which of the following statements about prudence is correct?
a) Assets and income must be b) The lowest possible values
understated while liabilities and should be applied to income and
expenses must be overstated. assets and the highest possible
values to expenses and liabilities.
c) Assets and income are not d) Understating of assets and

Prepared by: Dawood Shahid CA & CIMA Affiliate, CPA, MPhil, MBA, OCE
overstated while liabilities and income and overstating of
expenses are not understated. liabilities and expense are
desirable in preparing financial
statements.
210) On 30 June 2021, cashbook of Abbas Enterprises reflected a credit balance of Rs
167,000 and bank statement showed a credit balance of Rs 125,000. Which two of
the following situations can reconcile the balance?
a) Unpresented cheques of Rs b) Unpresented cheques of Rs
304,000 and bank charges of Rs 280,000 and bank charges of Rs
12,000 12,000
c) bank charges of Rs 12,000 and d) Unpresented cheques of Rs
uncleared cheques of Rs 304,000 432,000 and uncleared cheques
of Rs 140,000
211) Income arises from increase in assets or decrease in a liability resulting in:
a. Decrease in equity other than contribution from owner
b. Increase in equity other than contribution from owner
c. Increase in equity including contribution from owner
d. Decrease in equity including contribution from owner

212) Which two of the following would be considered as business transaction?


a. Business paid the electricity bill of previous year
b. Owner received the electricity bill of the owner residence
c. Owner paid the electricity bill of the owner residence from personal cash
d. Business received electricity bill of office premises which will be paid in
next year

213) Which two of the following would be considered as business transaction?


1. Bank set the limit of overdraft of Rs 10 million
2. Interest charged by bank on overdraft.
a) Both are correct b) None is correct
c) Only 1 d) Only 2
214) Which TWO of the following are not part of a complete set of the financial
statement?
a) Notes to financial statement b) Statement of financial position
c) Director report d) Auditor report

215) Which TWO of the following are not part of a complete set of the financial
statement?
a) Statement of cash flows b) Statement of financial position
c) Income tax return d) Auditor report

Prepared by: Dawood Shahid CA & CIMA Affiliate, CPA, MPhil, MBA, OCE
216) Total assets of a business would be charged as a result of:
a) Non-current assets being b) A supplier balance being paid in
purchased on cash cheque
c) Wages being paid in cash d) Non-current assets being
purchased on credit
217) A business commenced with capital in cash of Rs. 150,000. Inventory of Rs
100,000 was purchased for cash and half of inventory was sold at a mark-up of
20% on credit. Furniture of Rs. 40,000 was purchased on credit. What will be
accounting equation (A-L=C) after these transactions.
a) Rs. 190,000 - Rs. 40,000 = Rs. 150,000 b) Rs. 200,000-Rs 40,000=Rs 160,000
c) Rs. 150,000-Rs 40,000=Rs 110,000 d) Rs160,000-Rs 40,000=Rs 120,000
218) If during the accounting period the assets increased by Rs. 7 million, and the
owner’s equity decreased by Rs. 3 million, then the liabilities must have:
a) Increased by 10 million b) Increased by 4 million
c) Decreased by 4 million d) Decreased by 10 million
219) Which TWO of the following would be recorded in General journal?
a) Increase in provision of b) Advance received from customer
doubtful debts
c) Write off of a trade receivable d) Receipt of a written off balance
balance
220) Which of the following document would require a double entry?
a) Sales order b) Sales invoices
c) Delivery note d) Statement of account
221) Match each of the following transection with appropriate book of prime entry
Sales made on credit Sales day book
Sales made on cash Cash book
Return by credit customer Return inward journal
Purchase of non-current asset on credit General journal
222) A transection not recorded in any other books of original entry is recorded in:
a) Sales day book b) Purchase day book
c) Cash book d) General journal
223) Zee bought goods on credit from Shan traders. The goods were not according to
the required specification and therefore returned to ST. which document should
zee send to ST
a) Statement of account b) Credit note
c) Debit note d) Invoices
224) Which two of the following statement are correct regarding trial balances?
a) Opening stock as well as closing b) Trail balance is not a book of

Prepared by: Dawood Shahid CA & CIMA Affiliate, CPA, MPhil, MBA, OCE
stock appear in the trail balance prime entry
c) Every credit balance represent d) Return inward has debit
income balance
225) On 4 August 2021 goods were sold on credit for Rs. 125,000 with credit term of
3/10, n/20. On 8 August 2021 goods of Rs. 25,000 were returned by the buyer.
How much should the seller expect to receive if the buyer pays on 12 August 2021
a) Rs. 100,000 b) Rs. 121,250
c) Rs. 125,000 d) Rs. 97,000
226) Which two of the following would be appear in trail balance?
a) Gross profit b) Discount allowed
c) Trade discount received d) Acc. Depreciation
227) How would these assets will expire?
Prepaid rent Office supplies

Through use and consumption With the passage of time


Through use and consumption Trough use and consumption
With the passage of time Through use and consumption
With the passage of time With the passage of time
228) Consider the following statement:
i. Higher closing prepaid balances than previous year means that payment for the
year exceeds the expense for the year
ii. Higher closing accrued income balances means that receipt for the year exceeds
the income for the year.

Which of the following is/ are correct?


a) None is correct b) Only 1 is correct
c) Only 2 is correct d) Both are correct
229) On 1 February 2019 a loan of Rs 6,000,000 was taken from a bank for acquisition
of an office building. A portion of the building was rented out on 1 August 2019 at a
monthly rent of Rs 45,000. Interest is payable at 15% per annum on 31 January
each year and rent are received half yearly in advance.
What amounts of interest payable and unearned rent should be shown in statement of
financial income as on31 December 2019?
a) Interest payable Rs 900,000, b) Interest payable Rs 450,000,
unearned rent Rs 270,000 unearned rent Rs 45,000
c) Interest payable Rs 825,000, d) Interest payable Rs 825,000,
unearned rent Rs 45,000 unearned rent Rs 250,000

Prepared by: Dawood Shahid CA & CIMA Affiliate, CPA, MPhil, MBA, OCE
230) An entity received electricity bill for the month of June 20. The bill will be paid on
10 July 20. The entity financial year ends on 30 June. How would the payment be
recorded?
a) Debit utilities payable and credit b) Debit utilities payable and credit
electricity expense cash
c) Debit electricity expense and d) Debit electricity expense and
credit utilities payable credit cash
231) On 1-7-19 a business acquired a shop on rent which is payable quarterly in
advance. Payments are, made as follow:
Date Rupees
1 July 2019 75,000
30 September 2019 75,000
31 December 2019 75,000
31 March 2020 75,000
30 June 2020 75,000
Which will be the rent expense charged to P/L for the year ended 31-08-20
a) Rs 75,000 b) Rs 375,000
c) Rs 300,000 d) Rs 225,000
232) At 31 December 2019 an entity allowance for doubtful receivables amounted to
Rs. 48,000 which was 5% of the receivables at that date. At 31 December 2020
receivables totalled Rs 1,085,000. It was decided to write off Rs 53,000 of debt as
irrecoverable and based on past experience to keep the allowance for doubtful
receivables at 5% of receivables.
What should be the charge in P/L for the year ended 31 December 2020 for bad
and doubtful receivables.
a) Rs 59,250 b) Rs 56,600
c) Rs 49,400 d) Rs 46,750
233) At 31 December 2020 receivables totalled Rs. 785,000 before making the
following adjustment.
i. It was decoded to write off Rs 20,000 of bad debts as irrecoverable and create
specific allowance of 80% against balance of Rs. 75,000.
ii. General allowance for receivables is to be maintained at 5% of receivables.

Allowance for doubtful receivables as at 31-12-19 amount to 31000


What should be the allowance for doubtful receivables at December 2020
a) Rs 110,000 b) Rs. 109,500
c) Rs 95,500 d) Rs 94,500
234) Which is the purpose of maintaining an allowance for doubtful debt receivables.
a. Records irrevocable debts without taking them out of the books of an entity

Prepared by: Dawood Shahid CA & CIMA Affiliate, CPA, MPhil, MBA, OCE
b. Records the expense of irrevocable debts
c. Have an estimate of future irrecoverable debts
d. Matches the estimate cost future irrecoverable debts against the revenue earned
in giving rise to the potential irrecoverable debts

235) For reconciling bank balances unpresented cheques should be:


a. Deducted from balance of bank statement or added to balance of cash
book
b. Added to balance of bank statement or deducted from balance of cash book
c. Deducted from balance of bank statement and added to balance of cash book
d. Added to balance of bank statement and deducted from balance of cash book

236) Which of the following statement is correct?


a. Credit balance as per bank statement means a bank overdraft
b. Debit balance as per bank statement means a bank overdraft
c. Debit balance as per cash book means bank overdraft
d. credit balance as per cash book means an asset.

237) Determine the balance as per the bank statement using the following information.
RUPEES
Balance as per cash book 49100
Cheques received and deposited into the bank but not yet credited in the 4600
bank statement
Unpresented cheque 6300
Credit transfer appearing in the bank statement but not entered in the cash 3400
book

a) Rs 52,500 b) Rs 54,200
c) Rs 58,800 d) Rs 50,800
238) A business has an overhead absorption rate of RS 425 PER MACHINE HOUR
based on budgeted level of activity. Which is 12400 hours. In the period covered
by the budget actual machine hours worked were 2% more than the budgeted
hours and actual overhead expense incurred was RS. 5638900. What was the
absorbed OH for the period?
a) Rs 5,638,000 b) Rs 5,375,400
c) Rs 5,270,000 d) Rs 5,500,000
239) What entry should be recorded when actual OH expenditure is incurred
DEBIT CREDIT
COST OF SALE PRODUCTION OH
INVENTORY WIP PRODUCTION OH
PRODUCTION OH CASH

Prepared by: Dawood Shahid CA & CIMA Affiliate, CPA, MPhil, MBA, OCE
PRODUCTION OH COST OF SALE
240) Consider the following diagram:

Which of the following cost would be most likely to be illustrated by the


diagram given below?
a) Indirect material cost b) Direct material cost
c) Factory rent d) Labour cost
241) Which of the following best describes a period cost?
a. A cost that is identifies with a unit produced during the period is included in the
value of inventory. The cost is treated as an expense for the period when the
inventory is actually sold.
b. A cost that can be easily allocated to a particular period without the need for
arbitrary apportionment between periods.
c. A cost that relates to a time period is not included in the inventory
valuation. The entire cost is treated as an expense for the period
d. A cost that is incurred regularly every period e.g., every month or quarter.

242) Which of the following best describes a liability?


a) Someone who owes money to the b) A transaction to purchase goods
business for goods purchased on to be paid for immediate
credit. payment.
c) An amount owed by the d) All of these
business
243) Depreciation is best described as:
a) A means of estimating amount needed to replace the assets.
b) A means of spreading the net cost of non-current assets over their
estimated useful life.
c) A means of spreading the net cost of current assets over their estimated
useful life.
d) None of these.
244) As useful life decreases, depreciation value will
a) Increase b) Decrease
c) Constant d) All of these depending on
circumstances.

Prepared by: Dawood Shahid CA & CIMA Affiliate, CPA, MPhil, MBA, OCE
245) Which two of the following items would be appearing in analysis of expenses by
nature?
a) Salaries expense b) Admin expense
c) Distribution expense d) Finance cost
246) A liability is classified as current liability if it is paid;
a) Out of current assets b) Within 1 year or normal
operating cycle whichever is
longer
c) Out of non-current assets d) Within 1 year or normal operating
cycle whichever is shorter
247) The purpose of depreciation is to;
a) Recognize that asset lose value overtime
b) Write the asset down to its realizable value at the end of each period
c) Allocate the depreciable cost on a systematic basis over the asset’s useful
life
d) Accumulate a fund for asset replacement

248) If an asset is idle, then in which type of methods no depreciation is charged?


a) Straight line method b) Production units method
c) Both a and b d) Reducing balance method
249) Which of the following is not an asset that falls under the scope of IAS 16 PPE?
a. Assets held for the production or supply of goods or service
b. Tangible assets
c. Assets held for sale in the normal course of business
d. Assets expected to be used for more than one period

250) Alpha Trading Limited (ATL) used its own staff, assisted by contractors when
required, to construct a new warehouse for its own use. Identify the costs listed
below that cannot be capitalized.
a) Clearance of the site prior to commencement of construction
b) Professional surveyor fees for managing the construction work
c) ATL’s own staff wages for time spent working on construction
d) A proportion of ATL’s administration costs, based on staff time spent

251) A motor vehicle was purchased for Rs. 1,395,000 on 1 July 2018. It has an
estimated useful life of 5 year and residual value of Rs. 255,000. If the sum of digit
method is used for depreciation what will be the carrying amount of the motor
vehicle as at 30 June 2020
a) Rs 303,000 b) Rs 456,000
c) Rs 711,000 d) Rs 558,000

Prepared by: Dawood Shahid CA & CIMA Affiliate, CPA, MPhil, MBA, OCE
252) During the year 2019 an entity purchased a machine for Rs. 20 million to be used
for 6 years. Which of the following would represent residual value of the machine
in 2019?
a. Rs 4 million can be currently obtained from disposal of a 6-year-old similar machine
b. Rs 15 million can be currently obtained from disposal of the machine in present condition
c. Rs. 18 million can be obtained in 2025 from the disposal of the machine in present condition.
d. Rs. 7 million can be obtained in 2025 from disposal of a 6-year-old similar machine.

253) If a PPE remains idle for whole year, the depreciation expense for the year will be
NIL under:
a) Straight line method b) Units of production method
c) Reducing balance method d) All of the above method
254) Which of the following does not require Journal entry in periodic inventory method?
a) Goods retuned by a supplier b) Closing stock
c) Abnormal loss d) Normal loss
255) A company purchased a machine for Rs. 25 million having residual value of 2
million. The machine is being depreciated @ 15% using reducing balance method.
After two years, the company decided to change its depreciation estimate to
straight line method. The remaining useful life of the machinery is 5 years with
residual value of Rs. 3 million. The NBV (net book value) at third year is
approximately
Select the most appropriate answer
a) Rs 15.1 million b) Rs 12.1 million
c) Rs 11.5 million d) Rs 14.5 million
256) A company purchased a manufacturing plant for Rs. 25 million having residual
value of 3 million. The plant is being depreciated @ 20% per annum using
reducing balance method. What would be the NBV (net book value) of the plant
after third year?
Select the most appropriate answer
a) Rs 11.8 million b) Rs 14.3 million
c) Rs 12.8 million d) Rs 10 million
257) Which two of the following is included in cost of inventory?
a) Recoverable taxes paid on b) Normal wastages of material
purchases
c) Selling cost d) Fixed manufacturing
overheads
258) Bilal has started a trading business. He deals in goods where price is usually rising
over the time. Which of the following statement is correct regarding selection of
FIFO and average method in this case?
a) FIFO will result in lower reported b) Average method will give more

Prepared by: Dawood Shahid CA & CIMA Affiliate, CPA, MPhil, MBA, OCE
profit accurate profit

c) Avg. method will lead will higher d) Profit will be unaffected by the
cost of ending stock selection of inventory method

259) In preparing its financial statement for the current year an entity closing inventory
was understated by RS. 200000. What will be the effect of this error if it remains
uncorrected?
a. The current year profit will be overstated and next year profit will be understated
b. The current year profit will be understated and next year profit will be
overstated
c. The current year profit will be overstated but there will be no effect on next year
profit.
d. The current year profit will be understated but there will be no effect on next year
profit.

260) What is the effect on profit for the year and net assets when accrued income is not
recorded?
Profit for the year Assets
a) Overstated Overstated
b) Overstated Understated
c) Understated Overstated
d) Understated Understated
261) Which of the following is the correct entry of not-sufficient-fund (NSF)?
Answer: Receivables (Dr) Bank (Cr)
262) Which of the following is the correct entry of not-sufficient-fund (NSF)?
Answer: Receivables (Dr) Bank (Cr)
263) Statement of Comprehensive income is prepared for
a) Whole year b) Last day of Accounting year

264) Statement of Financial Position is prepared for


b) Whole year b) Last day of Accounting year

265) Seller received which note from buyer?


a) Debit note b) Credit Note.

266) Credit sales goes to which of the following books of prime entry?
a) Sales day Book

Prepared by: Dawood Shahid CA & CIMA Affiliate, CPA, MPhil, MBA, OCE
267) Credit note issued is recorded in which Books of Prime Entry?
a) Sales return Day book.

268) Debit note issued is recorded in which Books of Prime Entry?


a) Purchase return Day book.

269) Which of the following statement is correct?


(I) Debit notes issued are initially recorded in purchase day book.
(II) Sales invoices are recorded in sales day book.
a) Both are correct b) None is correct
c) Only 1 d) Only 2
270) which of the following statement is correct?
(I) Credit notes issued to customers is the source document for sales day book.
(II) Cheques and counterfoils are the source documents for cashbook.
a) Both are correct b) None is correct
c) Only 1 d) Only 2
271) In which Books of Prime Entry does Credit note received worth Rs. 800,000 would
be recorded?
a) Sales day book b) Purchase day book
c) Return outward Journal. d) Return inward journal
272) By which error totals of trial balance will disagree?
a) Sales made on credit had been b) Fixtures purchased by cheque
credited to account of wrong were not recorded at all.
customer
c) Fixtures purchased by cheque d) Sales made on credit had been
were not recorded as asset. debited to account of wrong
customer
273) What is the Journal entry of settlement discount?
Discount Allowed Dr receivables Cr.
274) A business is offering a settlement discount to customers. In which TWO of the
following situations, the revenue is required to be recorded at the full amount at the
time sale?
a) The customer is not expected to b) The customer is expected to pay
pay early and does not pay early and pays early as expected.
early as expected.
c) The customer is not expected to d) The customer is expected to pay
pay early but pays early. early and does not pay early.

Prepared by: Dawood Shahid CA & CIMA Affiliate, CPA, MPhil, MBA, OCE
275) Which of the following reasons require an entry in the bank account in your
general ledger?
(I) A cheque from one of your customers amounting to Rs. 24,000 was returned
by the bank as unpaid because the customer has gone bankrupt.
(II) A cheque amounting to Rs. 18,000 in the name of your brother was
erroneously credited by bank in your account.
a) Both are correct b) None is correct
c) Only 1 d) Only 2
276) Dishonour cheques are recorded on which side of the Cash book?
a) Receipt side b) Payment side
c) Both sides d) None of these
277) Replenishment in petty cash fund is recorded in
a) Cash book + petty cash b) Petty cash
c) Sales day book d) Purchase day book
278) Payment of expenses from petty cash fund are recorded in:
a) Cash book only b) Only Petty cash book
c) Cash book as well as petty cash d) General journal only.
279) The funds of petty cash book are recorded in
a) Cash book + petty cash b) Only Petty cash
c) Only cash book d) General ledger
280) Accrual for Receiver is Prepayment for Payer
a) True b) False
281) Petty cash book must follow imprest system
a) True b) False
282) Petty expenses may exceed imprest amount.
a) True b) False
283) What are the properties of trial balance?
1) It is a summary of debit and credit balances which are extracted from various
ledger accounts.
2) It is a summary of debit and credit balances.
3) The motive behind the preparation of Trial balance is to establish arithmetical
accuracy of the transactions recorded in the Books of Accounts.
4) It serves as evidence that the double entry system has complied duly
5) It facilitates the preparation of the financial statements.
284) What is materiality concept?
Answer

Prepared by: Dawood Shahid CA & CIMA Affiliate, CPA, MPhil, MBA, OCE
Materiality concept in accounting refers to the concept that all the material items should
be reported properly in the financial statements. Material items are considered as those
items whose inclusion or exclusion results in significant changes in the decision making
for the users of business information.
285) Which of the following explains why lenders are interested in financial statements
of a business?
a) Lender need information about b) Lender need information about
financial stability of business. profitability of business.
c) Lenders want to assess the d) All of these
entity’s capacity to pay interest
and repay loan on time.
286) what is cost of conversion?
Answer: Direct labour+ factory Overheads
287) sales are recorded in?
a) Sales day book b) Sales return day book
c) General ledger d) General Journal
288) Which of the following cost is capitalised?
a) Interest paid on loan b) Monthly electricity bill
c) Carriage paid on purchase of d) Annual motor vehicle tax.
plant
289) An entity imported a computer server machine having purchase price of Rs. 8.5
million. The entity further incurred the following cost:
Expenditure Amount
Import duties 8%
Non- refundable taxes 5% purchase price plus import duties
Refundable taxes 10% of purchase price plus import duties
Insurance in transit Rs. 240,000
Carriage inward Rs. 450,000
The server should be capitalised at:

a) Rs 10.295 million b) Rs 11.145 million


c) Rs 9.845 million d) Rs 10.055 million
290) Purchase of machinery for cash
a) Increase total assets b) Decrease total assets
c) Increases assets and liabilities d) Keeps total assets unchanged
291) Investment of cash into business results in.
a) Increase in cash and decrease in b) Increase in cash and increase
capital. in capital.
c) Increase in fees earned and an d) Decrease in cash and an

Prepared by: Dawood Shahid CA & CIMA Affiliate, CPA, MPhil, MBA, OCE
increase in capital. increase in capital.
292) Services rendered for cash will result in a/an
a) Increase in cash and a decrease b) Increase in cash and an
in capital. increase in fees earned.
c) Decrease in cash and an increase d) Increase in fees earned and an
in fees earned. decrease in capital.
293) If we issue amount to replenish petty cash book, then it is recorded in
a) General journal b) Debit side of cash book
c) Credit side of cash book d) General ledger.
294) Which of the following is business transaction?
a) Electricity bill paid of owner house. b) Wages being paid by cash
c) Hiring of salesman for business d) Selection of better quality
material for business
295) If we pay our supplier by bank overdraft, then it is
a) Decrease in asset and increase in b) Decrease in asset and decrease
liability in liability
c) Decrease in liability and d) Increase in liability and increase
increase in another liability in another liability
296) Which of the following is business transaction?
a) Electricity bill paid of owner house. b) Payment to supplier and
electricity bill of factory
c) Hiring of salesman for business d) Selection of better quality
material for business
297) Which of the following is NOT a business transaction?
a) Interest charged by bank on b) Appointment of worker at
overdraft. 20,000 wage per month
c) Payment to supplier and electricity d) Wages being paid by cash
bill of factory
298) In output method, Depreciation is charged on which basis?
a) Usage of asset basis b) Value of asset basis
c) Useful life basis d) All of these
299) What is consistency basis in accounting?
Answer: In accounting, consistency requires that a company's financial statements
follow the same accounting principles, methods, practices and procedures from one
accounting period to the next. This allows the readers of the financial statements to
make meaningful comparisons between years.

Prepared by: Dawood Shahid CA & CIMA Affiliate, CPA, MPhil, MBA, OCE
300) What is retail method?
Answer: t is often used in the retail industry for measuring inventories of large numbers
of rapidly changing items with similar margins for which it is impracticable to use other
costing methods. The cost of the inventory is determined by reducing the sales value of
the inventory by the appropriate percentage gross margin.
301) What is the entry of stock given in Charity under periodic system?
a) Debit: charity expense b) Debit: charity expense
Credit: Purchases Credit: inventory
c) Debit: charity expense d) Debit: charity expense
Credit: supplier Credit: purchases return
302) What is the entry of stock given in Charity under perpetual system?
a) Debit: charity expense b) Debit: charity expense
Credit: Purchases Credit: inventory
c) Debit: charity expense d) Debit: charity expense
Credit: supplier Credit: purchases return
303) What is the entry of dishonoured cheques?
a) Debit: Receivables b) Debit: Bank Account
Credit: bank Account Credit: Receivables
c) Debit: Receivables d) Debit: Payables
Credit: Cash Account Credit: bank Account
304) An asset was purchased for Rs. 1,000,000 with the down payment of Rs. 200,000
and bills accepted for Rs. 800,000/-.
What would be the effect on the total asset and total liabilities in the statement of
financial position?
a) Assets increased by Rs.800,000 and liabilities decreased by Rs.800,000
b) Assets decreased by Rs.800,000 and liabilities increased by Rs.800,000
c) ) Assets increased by Rs.1,000,000 and liabilities increased by Rs.800,000
d) Assets increased by Rs.800,000 and liabilities increased by Rs.800,000
305) The owner contributes his personal car to the business
a) Increase an asset, increase a b) Decrease an asset, decrease a
liability. liability.
c) Increase an asset, increase d) Decrease an asset, decrease
owner's equity owner's equity
306) When market prices are declining, which method give less profit?
a) FIFO b) Weighted average
c) Both of these d) None of these
307) The process of classifying amounts from books of prime entry to ledgers is called
a) Posting b) Recording

Prepared by: Dawood Shahid CA & CIMA Affiliate, CPA, MPhil, MBA, OCE
c) Summarizing d) Presentation
308) What are the limitations of trial balance?
Answer:
Both the debit and credit columns of the trial balance may be equal in spite of certain
mistakes of omissions and principles. These errors may be mentioned as under:
1) Errors of omission in the books of original record
2) Errors of principle (posting on right side of wrong account nature wise)
3) Compensating errors (same amount effect on debit and credit side)
4) Incorrect account in the original books (on both sides)
5) Posting to wrong account (but on right side)

309) The process of transferring transaction from journal to ledgers is called?


a) Journalizing b) Summarizing
c) Posting d) Analysing
310) What is the entry of discount allowed?
a) Debit: Discount Allowed b) Debit: Discount Allowed
Credit: Sales Credit: Receivables
c) Debit: Discount Allowed d) Debit: Discount Allowed
Credit: purchases Credit: Payables
311) Discount received is recognised as:
e) Reduction from purchases in f) Expense in profit or loss
general journal. statement
g) In purchase return journal h) Deduction in purchase day book
312) Bad debts recovered is recognised as:
a) Increase in allowance b) Other income
c) An expense d) None of these
313) Payment of goods in transit is recorded in BRS
a) Added in Bank statement balance b) Deducted from Bank statement
or deducted from cashbook balance or added in cashbook
balance balance
c) Is never adjusted in BRS d) None of these.
314) Royalty paid is
a) Direct expense b) Indirect expense
c) Capital expenditure d) Admin expense
315) Dishonoured cheque is recorded in
a) Receipt side of cashbook b) Payment side of cashbook
c) Both sides of cashbook d) None of these

Prepared by: Dawood Shahid CA & CIMA Affiliate, CPA, MPhil, MBA, OCE
316) How would dishonour of customers cheque be recorded in cashbook?
Select the most Appropriate Answer
a) Recorded on Receipt as well as b) Not recorded in cashbook
payment side of cashbook
c) Recorded on payment side only d) Recorded on receipt side only.
317) The following data pertains to Alpha Traders(AT).
Closing balance of trade debtors Rs. 1,160,000
Opening balance of allowance for doubtful debts Rs. 90,000
Debtors need to be written off. Rs. 110,000
AT’s policy is to maintain the allowance for doubtful debts @10%. The closing
balance of allowance for doubtful debts would be:
Select the most appropriate answer:
a) Rs 96,000 b) Rs 107,000
c) Rs 105,000 d) Rs 116,000
318) Prudence concept means that
1. Expenses and liabilities must not be understated
2. Incomes and assets must not be overstated.
a) Only 1 is correct b) Only 2 is correct
c) Both are correct d) None of these
319) Which of the following transaction will affect the receivable account?
a) Bad debts expense b) Allowance for bad debts
c) Purchase return d) Trade discount
320) Increase in liability and increase in capital results in
a) Asset Increase b) Assets Decrease
c) No impact d) None of these.
321) Which of the following elements have same nature?
a) Assets and liabilities b) Capital and liabilities
c) Assets and capital d) Capital and expenses
322) Which TWO of the following elements have same nature in trial balance?
a) Sale and return inwards b) Capital and accumulated
depreciation
c) Sale and return outwards d) Purchase and return outward
323) A trial balance consists of:
a) Two column financial statement intended for distribution to interested parties
outside the business.
b) A two column schedule stating names and balances of all ledger
accounts.

Prepared by: Dawood Shahid CA & CIMA Affiliate, CPA, MPhil, MBA, OCE
c) A two column schedule of all debit and credit entries posted to ledger
accounts
d) A two column schedule of all debits and credits made in journal entries.
324) If credit side< debit side in asset account, then difference is
a) Positive balance b) Negative balance
c) Credit balance d) None of these
325) Find depreciation expense in third year of an asset from using the following data:
Cost of asset= 25 million, residual value=2 million, reducing balance method having rate
of 20%
After two years’ company shifted its method to straight line with revised residual value 3
million and remaining useful life 6 years.
326) What are the generally accepted accounting principles in Pakistan?
a) IFRSs b) Companies Act 2017.
c) Both a and b d) None of these
327) Son received money Rs 1,000,000 from his father. Is this a business transaction?
a) Yes b) No
328) Equity is residual interest in the assets of business after deducting current
liabilities. Is this definition correct?
a) Yes b) No
329) In which book of prime entry drawings is initially recorded?
a) Purchase day book b) Sales day book
c) Cash book d) General Journal
330) Which of the following is taken as a test of arithmetical accuracy?
a) General Journal b) General ledger
c) Books of prime entry d) Trial balance
331) In the year to 31 December 2018, Saira received Rs. 50,800 rental income. The
amounts of rent received in advance and due in arrears were as follows:
31 Dec 2018 31 Dec 2017
Rs. Rs.
Rent received in advance 4,000 3,000
Rent due in arrears (accrued) 2,500 1,700
What figure for rental income should be recorded in the statement of profit or loss for
the year ended 31 December 2018?
a) Rs. 50,800 b) Rs. 50,600
c) Rs. 54,500 d) Rs. 56,000

Prepared by: Dawood Shahid CA & CIMA Affiliate, CPA, MPhil, MBA, OCE
332) Total rent of Rs. 150,000 were received during the year ended 31 December 2019.
In this respect opening and closing balances are as under:
31 Dec 2018 31 Dec 2017
Rs. Rs.
Rent received in advance (unearned) 16,000 10,000
Rent due in arrears (accrued/ rent receivable) 13,000 17,000
What figure for rental income should be recorded in the statement of profit or loss for
the year ended 31 December 2019?
a) Rs. 144,000 b) Rs. 140,000
c) Rs. 152,000 d) Rs. 156,000
333) ABC Limited receives advance rent from its tenant of Rs.1 million on 31st
December in respect of office rent for the following year.
ABC Ltd. has an accounting year end of 31st December. What accounting entry is to be
passed in this year?
a) Cash (debit) = Rs.1 million and Advance rent – liability (credit) = Rs.1 million
b) Cash (debit) = Rs.1 million and Accrued rent (credit) = Rs.1 million
c) Cash (debit) = Rs.1 million and Rent income (credit) = Rs.1 million
d) None
334) Doubtful debt allowance account is
a) Asset account b) Liability account
c) Contra asset account d) Expense account
335) Maaz return goods to Saad which he bought on credit. How is this recorded in
Saad’s books?
a) Accounts receivable Dr, b) sales return Dr,
sales return Cr Accounts receivable Cr
c) purchase return Dr d) accounts receivable Dr
accounts receivable Cr purchase return Cr
336) Maaz return goods to Saad which he bought on credit. How is this recorded in
Maaz’s books?
a) Accounts receivable Dr, b) sales return Dr,
sales return Cr Accounts receivable Cr
c) purchase return Dr d) accounts payable Dr
accounts receivable Cr purchase return Cr
337) What is the difference between net asset and total asset?
Total Assets= equity + total Liabilities
Net Asset= Total Assets – Total liabilities = equity
338) What is cost unit?
Answer

Prepared by: Dawood Shahid CA & CIMA Affiliate, CPA, MPhil, MBA, OCE
A cost unit is defined as “a unit of quantity of product, service, or time (or a combination
of these) in relation to which costs may be ascertained or expressed.” In other words, a
cost unit is a standard or unit of measurement of the goods manufactured or services
rendered.
Electric Company: Cost unit per unit
Furniture Industries: Cost unit per number
Brick Industries: Cost unit per 1000 bricks
Oil Companies: Cost unit per litre
339) Bank correctly record payment of cheque of Rs. 236 while in cash book it is written
as Rs. 263. What should be done to correct it?
a) Add Rs. 27 in BRS b) Deduct Rs. 27 from BRS
c) Add Rs. 27 in cash book d) Deduct Rs. 27 from cashbook
340) Which one is incorrect?
1. All items of property plant and equipment must be depreciated
2. Depreciation charged in straight line method when asset is available for use
a) Both are correct b) None is correct
c) Only 1 d) Only 2
341) Which of the following is correct?
1. The balance of allowance for doubtful debts records irrecoverable debts without
taking them out of the books
2. The balance of bad debts recovered will carry down to next accounting period
a) Both are correct b) None is correct
c) Only 1 d) Only 2
342) Which of the following is not in the trial balance of perpetual inventory system?
a) Cost of goods sold b) Inventory
c) Sales return d) Purchase return
343) Direct cost is?
a) Directly attributable to a cost b)
unit
c) d)
344) In an account if credit side < debit side then the resulting balance is:
a) Debit b) Positive
c) Negative d) Credit
345) Which of the following is/ are correct?
1. Credit entries decrease liabilities and increase income.
2. Credit entries decrease expenses and increase assets

Prepared by: Dawood Shahid CA & CIMA Affiliate, CPA, MPhil, MBA, OCE
a) Both are correct b) None is correct
c) Only 1 d) Only 2
346) Balance of which of the following groups of item would appear on opposite sides of
a trial balance?
a) Inventory and drawings b) Carriage in and carriage out
c) Sales and return outwards d) Trade receivables and return
outwards.
347) Balance of which TWO of the following groups of item would appear on same
sides of a trial balance?
a) Capital and accumulated depreciation b) Sales and return outwards
c) Sales and return inwards d) Purchase and return outward
348) Which of the following statements are correct?
1. Sum of digits method and reducing balance method gives higher depreciation in
early years
2. Straight line method and sum of digit method compute depreciation over
economic life
a) Both are correct b) None is correct
c) Only 1 d) Only 2
349) Which of the following correctly describes over absorption?
a) Actual > absorption b) Absorption > actual
c) Budget > actual d) Actual > Budget
350) Which two of the following transactions will affect the assets and liabilities of the
business at the same time?
a) Purchase of office furniture on b) Credit sale of inventory
credit
c) Receipt of cash from customer d) Repayment of principal amount
of a loan
351) Why is it necessary to account for accrued expenses?
a) So that current liabilities are not b) So that current assets are not
overstated understated
c) So that profit is not understated d) So that profit is not overstated
352) Which of the following entry should be made at year-end for accrued expenses?
a) Debit: Expenses b) Debit: Accrued expenses
Credit: Accrued expenses Credit: Expenses
c) Debit: prepaid expenses d) Debit: Expenses
Credit: expenses Credit: prepaid expenses

Prepared by: Dawood Shahid CA & CIMA Affiliate, CPA, MPhil, MBA, OCE
353) Which of the following reversal entry should be made at start of next year for
accrued expenses?
a) Debit: Expenses b) Debit: Accrued expenses
Credit: Accrued expenses Credit: Expenses
c) Debit: prepaid expenses d) Debit: Expenses
Credit: expenses Credit: prepaid expenses
354) Which of the following entry should be made at year-end for prepaid expenses?
a) Debit: Expenses b) Debit: Accrued expenses
Credit: Accrued expenses Credit: Expenses
c) Debit: prepaid expenses d) Debit: Expenses
Credit: expenses Credit: prepaid expenses
355) Which of the following reversal entry should be made at start of next year for
prepaid expenses?
a) Debit: Expenses b) Debit: Accrued expenses
Credit: Accrued expenses Credit: Expenses
c) Debit: prepaid expenses d) Debit: Expenses
Credit: expenses Credit: prepaid expenses
356) What is an asset?
a) An asset is a present economic b)
resource controlled by the
entity as a result of past event.
c) d)
357) Which of the following is correct?
1. The good debts do not require any special accounting treatment unlike bad and
doubtful debts
2. The doubtful debts must stay in the accounting records so that the business
continues to chase payments
a) Both are correct b) None is correct
c) Only 1 d) Only 2
358) Which of the following is correct?
1. An asset is a resource controlled by the entity.
2. Equity is the residual interest in the assets of the entity after deducting all its
liabilities.
a) Both are correct b) None is correct
c) Only 1 d) Only 2
359) Which of the following equation is correct for calculating cost of goods
manufactured?
a) =Prime Cost + Opening inventory (raw material) – Closing inventory (raw material)
b) = Prime Cost + Opening inventory (WIP) – Closing inventory (WIP)

Prepared by: Dawood Shahid CA & CIMA Affiliate, CPA, MPhil, MBA, OCE
c) =Factory Cost + Opening inventory (raw material) – Closing inventory (raw
material)
d) = Factory Cost + Opening inventory (WIP) – Closing inventory (WIP)
360) Raw materials which are incorporated into cost of goods manufactured but are not
easily identifiable to the goods being made would be known as:
a) Manufacturing overheads. b) Direct material.
c) Work in progress. d) Direct overheads.
361) Prime cost means
a) Sum of direct material and factory overhead costs
b) Sum of direct material and selling costs
c) Sum of indirect material and direct labour costs
d) Sum of direct material and direct labour costs
362) Which of the following correctly describes a Prime cost?
a) All direct costs b) Production overheads
c) Direct labour cost+ direct d) Indirect labour
expenses
363) Which of these TWO transactions would cause immediate change in equity:
a) Delivery truck bought on credit b) Borrowing from bank @12%
c) Repayment of loan from d) Write down of inventory from
personal cash to save interest cost to its NRV
expense.
364) Which of the following statements regarding inventory valuation is correct?
a) All inventory items should be b) The purchase price of items
valued at cost. which have been held for the
longest period is an acceptable
method for valuing inventory.
c) Inventory valuation should exclude d) All inventory items should be
profit which has not yet been valued at anticipated selling price
earned less any cost which will be
incurred.
365) Which of the following statements regarding are correct?
(I) Storage cost of finished goods is not included in the cost of inventory
(II) Normal capacity of production takes into account the loss of capacity resulting
from planned maintenance.
Select the most appropriate answer.
a) Only I b) Only II
c) Both correct d) None correct
366) What is the objective of accounting?
Answer

Prepared by: Dawood Shahid CA & CIMA Affiliate, CPA, MPhil, MBA, OCE
the main objective of financial accounting is to accurately prepare an organization's
financial accounts for a specific period, otherwise known as financial statements.
Other objectives are as follows
 The following are the main objectives of accounting:
 To maintain full and systematic records of business transactions:
 To ascertain profit or loss of the business:
 To depict financial position of the business:
 To provide accounting information to the interested parties:
367) What is the source document of sales day book and cashbook?
Answer: Sales invoices issued to customers
368) What is the source document of cashbook?
Answer: cheques, counterfoils, bank statements, receipts, etc.
369) Which of the following is not a source document?
a) Sale invoice b) Purchase invoice
c) Ledger d) Bank statement.
370) Credit sales return recorded in _______ ledger.
Answer: General ledger
371) What is the formula of straight line method for depreciation expense?
372) Which of the following correctly describes over absorption?
a) Rs 1.8 million b) Rs 3.6 million
c) Rs 0.2 million d) Rs 1.6 million
373) Following summarized trial balance as at 31 December 2015 pertains to Moon
Trading (MT) who deals in office machines:
Debit Credit
Rs. In million
Other income 15
Review of other income revealed the following information:
On 1 August 2015, MT received an amount of Rs. 1.8 million as 50% advance
against a maintenance contract covering the period from 1 September 2015 to 31
May 2016 and was credited to other income. The balance amount would be paid
on completion of the contract.
a) Rs 1.8 million b) Rs 3.6 million
c) Rs 0.2 million d) Rs 1.6 million
374) Which of the following correctly describes over-absorption?
a) Rs 1.8 million b) Rs 3.6 million
c) Rs 0.2 million d) Rs 1.6 million

Prepared by: Dawood Shahid CA & CIMA Affiliate, CPA, MPhil, MBA, OCE
375) Maria Enterprises (ME) sold goods of Rs. 10,000 to Zahra Traders (ZT) on 8th
August 2021 to be paid on 31st August 2021. However, if ZT pays within 10 days,
it will be entitled to 4% cash discount and will have to pay only Rs. 9,600.
How the above transactions will be treated in the books of ME on 8th August and
on the date of payment if ME expected that ZT will not pay within 10 days and ZT
actually paid on 31st August.

Prepared by: Dawood Shahid CA & CIMA Affiliate, CPA, MPhil, MBA, OCE
Question-1
Following is the trial balance of Salman for the year ended 30 June 2014:

Rs. in‘000’ Rs.in‘000’


Debit Credit
Drawings 30,500 Salman’s capital 300,000
Trade Debtors 109,420 Creditors 104,724
Goodwill 20,000 Loan taken from bank 40,000
Interest on Bank loan 800 Bad debts provision as on 1 900
July 2013
Cash in hand 728 Sales 353,300
Plant & Machinery-WDV 45,000 Interest received 11,930
Factor building-WDV 42,400 Bills payable 23,150
Sales return 10,000 Accrued expenses 3,460
Cash at bank 13,512 Other income 3,100
Power and utility charges 6,500
Trade discount 2,432
Purchase less return 330,530
Salaries and wages 40,400
Rent and rates 6,992
Insurance 2,000
Advertisement 4,512
Opening stock 127,762
Carriage inward 10,420
Bills receivable 32,526
Miscellaneous expenses 4,130
840,564 840,564
Additional information:
(i) On 1 July 2013 a machine was purchased for Rs. 500 thousand. However, this
cost has wrongly been posted to the purchases account.
(ii) Salman’s son works as the head of administration and received a salary of Rs.
150 thousand per month, which has been included in drawings.
(iii) Insurance premium was paid in advance to the extent of Rs. 650 thousand.
(iv) Depreciation is to be charged on written down value (WDV) as follows:
Plant and machinery 10%
Factory and building 5%
(v) Provision for bad debts is maintained at 2% of trade debtors.
(vi) Closing stock as on 30 June 2014 amounted to Rs. 237,500 thousand.
Required:

Prepared by: Dawood Shahid CA & CIMA Affiliate, CPA, MPhil, MBA, OCE
Prepare statement of comprehensive income (Trading and Profit and Loss account) for
the year ended 30 June 2014 and statement of financial position (Balance Sheet) as at
30 June 2014. (20)
Question-2
Following is the summarized trial balance of Fortune Traders (FT) for the year ended
30 June 2016:
Debit Credit
Rs. In “000”
Buildings - cost 3,700
Buildings - accumulated depreciation as at 1 July 2015 1,436
Plant and machinery - cost 6,650
Plant and machinery - accumulated depreciation as at 1 July 2015 2,414
Stock-in-trade as at 1 July 2015 1,045
Purchases 16,000
Purchase returns 220
Sales 28,900
Sales returns 90
Capital as at 1 July 2015 3,000
Selling and administration expenses 5,855
12% loan payable 5,000
Trade receivables 3,600
Provision for doubtful debts as at 1 July 2015 180
Prepayments and other receivables 380
Trade and other payables 2,080
Cash and bank balances 5,850
Suspense account 60
43,230 43,230
Additional information:
(i) FT depreciates its fixed assets from the month of addition. Depreciation is to be
charged on written-down value (WDV) as follows:
Plant and machinery 10%
Building 5%
(ii) On 1 March 2016, FT paid an advance of Rs. 330,000 for purchase of a machine
and debited it to plant and machinery. The machine was delivered on 1
September 2016.
(iii) Closing inventory was valued at Rs. 1,560,000. This included goods costing Rs.
35,000 returned by a customer on 30 June 2016 but not yet accounted for. These
goods were earlier sold at cost plus 40%.
(iv) The loan was acquired on 1 January 2016 and the principal amount is repayable
in eight equal half yearly instalments commencing from 1 January 2017. Interest
is payable half yearly on 1 January and 1 July each year.

Prepared by: Dawood Shahid CA & CIMA Affiliate, CPA, MPhil, MBA, OCE
(v) Selling and administration expenses include fire insurance premium amounting to
Rs. 430,000 and Rs. 240,000 paid for office and owner's personal premises
respectively. The policies are valid up to 31 December 2016.
(vi) Rent and salaries amounting to Rs. 137,000 and Rs. 89,000 respectively are to
be accrued at 30 June 2016.
(vii) At 30 June 2016, the provision for doubtful debts is to be reduced by Rs. 30,000.
(viii) Suspense account represents an error which occurred when a credit note of Rs.
30,000 received for goods returned to a supplier was mistakenly posted as credit
to trade payable account.
Required:
(a) Prepare a statement of comprehensive income for the year ended 30 June 2016.
(08)
(b) Prepare a statement of financial position as at 30 June 2016. (09)
Question 3:
Azam owns a retail outlet with the name Azam Autoparts Store. The trial balance as at
30 June 2015 is as follows:
Debit Credit
Rs. In ‘000’
Shop equipment- net 5,880
Motor vehicles- net 3,820
Stock as at 1 July 2014 14,500
Prepaid rent 2,000
Bank 8,500
Cash 50
Bank loan 5,050
Creditors 2,500
Accrued expenses 60
Capital 27,883
Sales 131,943
Purchases 105,950
Utility expenses 1,060
Vehicle expenses 1,205
Interest expenses 600
Renovation expenses 5,500
Salaries and wages 10,250
Depreciation expenses 2,075
Operating expenses 6,046
167,436 167,436
Additional information:
(i) Closing stock as at 30 June 2015 was Rs. 24,090 thousand.
(ii) Depreciation on fixed assets is charged at the rate of 20% on reducing balance
method.

Prepared by: Dawood Shahid CA & CIMA Affiliate, CPA, MPhil, MBA, OCE
(iii) Shop equipment was purchased on 1 July 2014 for Rs. 1,400 thousand. No
depreciation has been charged on it.
(iv) Renovation expenses represent cost of fixtures purchased for the shop.
(v) Rent is payable on yearly basis on 30 September. The expired rent till 30
September 2014 is included in operating expenses.
(vi) Bank loan was received on 1 July 2014. Interest payable for the month of June
2015 has been credited to the loan account.
(vii) Azam withdrew goods costing Rs. 4,000 thousand for personal use during the
year. However, no entry was made to record the withdrawal of goods.
(viii) Operating expenses include annual payment of fire insurance for shop and
personal expenses of Azam amounting to Rs. 725 thousand and Rs. 215 thousand. Fire
insurance policy would expire on 30 November 2015.
Required:
Prepare statement of comprehensive income for the year ended 30 June 2015 and
statement of financial position as at 30 June 2015. (20)
Question 4:
Following information has been extracted from the records of Falcon Traders (FT) for
the year ended 31 December 2013:
Non-current assets Rupees
Freehold land 100,000
Building 610,000
Plant and machinery 330,000
Office equipment 115,000
Current assets
Stock 183,000
Debtors 177,000
Bank 45,000
Non-current liabilities
10% long term loan from 190,000
bank
Current liabilities
Trade creditors 75,000
Creditors for office 35,000
expenses
Non-current assets Rupees
Freehold land 100,000
Building 610,000
Following information is available for the year 2014:
(i) Cash sales made during the year amounted to Rs. 375,000. Credit sales were
75% of the total sales and the amount collected from debtors was Rs. 1,035,000.
Provision for doubtful debts has to be made equal to 5% of the debtors’ balance
at the end of 2014. The provision for doubtful debts at the end of 2013 was nil.

Prepared by: Dawood Shahid CA & CIMA Affiliate, CPA, MPhil, MBA, OCE
(ii) Payments made to trade creditors amounted to Rs. 641,450. Credit purchases
were Rs. 664,950 whereas cash purchases were 35% of the total purchases.
(iii) FT paid Rs. 38,000 against salaries, Rs. 30,000 against office expenses and Rs.
45,000 against selling expenses.
(iv) Cash discount allowed to customers during the year amounted to Rs. 21,000
whereas discount received from suppliers was Rs. 13,000.
(v) On 1 July 2014 FT sold one of the old machinery at a loss of Rs. 15,000. On 31
December 2013 this old machinery had a book value of Rs. 54,800. On 1
October 2014 a new machine Z was purchased in its place at a cost of Rs.
164,800.
(vi) On 1 January 2014, office equipment was sold at its book value for Rs. 20,000.
(vii) Depreciation is to be provided on building at 10%, plant and machinery at 15%
and office equipment at 20% per annum on their book value.
(viii) Interest on long term loan at 10% per annum together with part payment of the
principal sum of Rs. 25,000 was made on 31 December 2014.
(ix) On 31 December 2014 closing stock was Rs. 90,000 and creditors for office
expenses were Rs. 28,000.
Required:
(a) Prepare a statement of comprehensive income for the year ended 31 December
2014. (09)
(b) Prepare a statement of financial position as at 31 December 2014. (11)
Question-5
Following summarised trial balance as at 31 December 2015 pertains to Moon Trading
(MT) who deals in office machines:
Debit Credit
Rs. In million
Fixed assets at cost 150
Accumulated depreciation 45
Trade debtors 200
Provision for doubtful debts 6
Inventory 410
Prepayments, advances and other receivables 9
Cash and bank balances 5
Capital 50
Bank loan 160
Trade creditors 320
Accruals and other payables 25
Sales revenue 2,840
Purchases 2,141
Selling and administration expenses 540
Interest on bank loan 8
Bank charges 2
Other income 15

Prepared by: Dawood Shahid CA & CIMA Affiliate, CPA, MPhil, MBA, OCE
Suspense account 4
3,465 3,465
Additional information:
(i) Inventory is valued under the FIFO method. Value of inventory as at 31
December 2015 amounted to Rs. 530 million.
(ii) During stock taking, it was noted that:
 A photocopy machine appearing in inventory is being used in the office since
1 October 2015. The cost of the machine is Rs. 0.78 million.
 Four computers were found to be short. It was discovered that these
computers costing Rs. 0.32 million had been issued for use by the proprietor’s
family members.
 Items costing Rs. 4.5 million were returned to a supplier on 31 December
2015. The return was recorded in January 2016.
 Items having invoice value of Rs. 2.4 million sold on credit were returned by a
customer on 31 December 2015. A credit note was issued and recorded by
MT on 12 January 2016. The profit margin on these items was 20% of cost.
(iii) Depreciation is charged on all fixed assets at 20% per annum under the straight
line method.
(iv) A recovery of Rs. 3.5 million against debts written off in prior years was credited
to trade debtors. MT’s policy is to provide doubtful debts at 3% on year-end
balance.
(v) Review of other income revealed the following information:
 Services for certain contracts amounting to Rs. 1.2 million were rendered in
December 2015 but invoices thereof were processed in January 2016.
 On 1 August 2015, MT received an amount of Rs. 1.8 million as 50% advance
against a maintenance contract covering the period from 1 September 2015
to 31 May 2016 and was credited to other income. The balance amount would
be paid on completion of the contract.
(vi) Office supplies purchased during the year are directly debited to expenses.
Unused office supplies at beginning and close of the year amounted to Rs. 0.45
million and Rs. 0.6 million respectively.
(vii) The bank loan was acquired on 1 April 2015. The principal amount is repayable
in five equal annual instalments on 31 March each year. Interest is payable at
10% per annum on six monthly basis and is recorded at the time of payment.
(viii) Suspense account represents a commission (net of bank charges amounting to
Rs. 0.25 million) received on 1 September 2015.
Required:
(a) Prepare a statement of comprehensive income for the year ended 31 December
2015. (11)

Prepared by: Dawood Shahid CA & CIMA Affiliate, CPA, MPhil, MBA, OCE
(b) Prepare a statement of financial position as at 31 December 2015. (09)
Question-6
Following is the summarised trial balance of Rainbow Lights (RL) for the year ended
31 December 2016:
Debit Credit
Rs. In million
Capital – 1 January 2016 70
Drawings 50
Fixed assets – cost 270
Accumulated depreciation 150
Closing inventory 170
Trade debtors 400
Provision for doubtful debts 12
Prepayments and other receivables 45
Cash and bank 20
10% Long-term loan 120
Trade creditors 240
Accruals and other payables 28
Sales 750
Cost of sales 304
Selling and administration expenses 146
Financial charges 10
Miscellaneous income 45
1,415 1,415
Additional information:
(i) RL uses perpetual inventory method to record its inventory. During the physical
inventory count carried out on 31 December 2016, following matters were noted:
 Inventory shortages amounted to Rs. 2 million which is considered to be normal.
 Goods costing Rs. 15 million were damaged in fire and have no sales value.
 Goods costing Rs. 1 million were withdrawn by the owner for his personal use
but no adjustment was made in the books.
 Goods sold on credit for Rs. 7 million were returned but have not been accounted
for. These goods were sold at cost plus 40%.
(ii) Goods sold on credit at a trade discount of 5% were recorded at gross amount of
Rs. 20 million.
(iii) Rs. 2 million were recovered in full and final settlement of an old outstanding
balance of Rs. 3 million which had been written-off last year. The amount
recovered was credited to trade debtors account.
(iv) RL maintains a provision for doubtful debts at 3% of the year-end balance.

Prepared by: Dawood Shahid CA & CIMA Affiliate, CPA, MPhil, MBA, OCE
(v) Miscellaneous income includes Rs. 12 million received against an annual
maintenance contract expiring on 30 April 2017.
(vi) Annual rent amounting to Rs. 24 million was paid in advance on 1 October 2016
and charged as an expense.
(vii) Long-term loan was acquired on 1 February 2016 and is repayable in 2020.
Interest thereon is due semi-annually on 1 August and 1 February each year.
Interest is charged to expenses at the time of payment.
(viii) An equipment costing Rs. 8 million was purchased on 1 September 2016 against
advance payment. The equipment was not used and returned on 31 December
2016. The supplier agreed to set-off the cost of the equipment against the
amount payable by RL. The return as well as reversal of depreciation is yet to be
recorded in the books.
(ix) Depreciation on fixed assets is charged at 15% per annum from the month of
addition to the month prior to disposal using reducing balance method.
Required:
(a) A statement of comprehensive income for the year ended 31 December 2016. (10)
(b) A statement of financial position as at 31 December 2016. (11)
Question-7
Following is the summarised trial balance of Home Appliances (HA) for the year ended
30 June 2017:
Debit Credit
Rs. in “000”
Fixed assets – cost 35,000
Accumulated depreciation – 1 July 2016 8,000
Inventory – 1 July 2016 8,200
Trade receivables 12,500
Provision for doubtful receivables – 1 July 2016 750
Prepayments and other receivables 2,350
Cash and bank balances 1,300
Capital 25,500
Drawings 4,500
Trade payables 10,200
Accruals and other payables 5,310
Sales 101,120
Return inwards 3,000
Purchases 70,000
Wages and salaries 5,900
Discounts 1,600
Utility charges 2,350
Carriage inwards 1,360
Running and maintenance expenses 3,290

Prepared by: Dawood Shahid CA & CIMA Affiliate, CPA, MPhil, MBA, OCE
Financial charges 580
Miscellaneous income 110 110
Suspense account 940
151,930 151,930
Additional information:
(i) Cost of closing physical inventory was Rs. 9,000,000. However, goods returned
by a credit customer on 30 June 2017 were received after completion of the
count, therefore, not included in the inventory. These goods had been sold for
Rs. 400,000 at cost plus 25%. However, the goods are damaged and can be sold
at normal price after a repair cost of Rs. 90,000. The return has not yet been
accounted for.
(ii) Suspense account represents an online transfer of funds by a customer who
availed 5% discount on settlement of the pending amount by 30 June 2017. The
amount was credited by the bank net of its charges of Rs. 10,000.
(iii) HA maintains a 5% provision against trade receivables.
(iv) Running and maintenance expenses totalling Rs. 110,000 were incurred in June
2017. Related invoices were received and accounted for in July 2017.
(v) A machine was rented-out to Zee Trader (ZT) on 1 October 2016. Annual rent of
Rs. 120,000 received in advance is included in ‘Accruals and other payables’.
However, on 16 June 2017, the machine was sold to ZT for Rs. 450,000. The
sale proceed net of rent adjustment was received in July 2017. The disposal has
not yet been accounted for.
The machine had been purchased on 1 January 2016 and had written down
value of Rs. 540,000 as at 1 July 2016.
(vi) HA depreciates its fixed assets at 20% per annum from the month of addition to
the month prior to disposal using reducing balance method. There were no
additions/disposals during the year except as mentioned above.
Required:
(a) Prepare statement of profit or loss for the year ended 30 June 2017. (11)
(b) Prepare statement of financial position as at 30 June 2017. (09)
Question-8
Following is the trial balance of Tulip Enterprises (TE) for the year ended 31
December 2017:
Description Debit Description Credit
Cash and bank balances 2,320 Trade payables 3,250
Trade receivables 4,400 Accruals and other payables 1,320
Stock-in-trade 31-12-2017 3,900 Provision for doubtful 220
receivables
Prepayments and advances 1,740 Accumulated depreciation 4,630
Property, plant & equipment – 12,500 12%Long-term loan 5,150

Prepared by: Dawood Shahid CA & CIMA Affiliate, CPA, MPhil, MBA, OCE
cost
Drawings 490 Capital 6,000
Cost of sales 23,580 Sales 35,230
Salaries and wages 2,610 Miscellaneous income 940
Fuel and power 450
Bad debt expense 230
Rent and insurance 2,900
Repair and maintenance 920
Financial charges 700
56,740 56,740
Additional information:
(i) While carrying out the physical inventory count at year-end, following matters
were identified:
 Goods costing Rs. 1,000,000 were slightly defective. These can be sold for
Rs. 1,130,000 after incurring a cost of Rs. 200,000.
 Goods costing Rs. 670,000 purchased on credit were returned to a supplier
on 28 December 2017 but the return was not recorded in the books.
(ii) A machine costing Rs. 450,000 was received on 1 October 2017 against 100%
advance payment. The advance has not yet been adjusted due to non-receipt of
the invoice.
(iii) On 1 October 2017, 50% advance received for an annual maintenance contract
of Rs. 480,000 was credited to miscellaneous income. Remaining amount would
be received at the end of the contract. Services are rendered evenly throughout
the contract period.
(iv) Maintenance services for Rs. 150,000 were rendered in December 2017 but
income has been recorded in January 2018 on receipt of the amount.
(v) Interest on the loan is paid in arrears on 1 April and 1 October each year. Interest
accrued for the quarter ended 31 December 2017 has been credited to loan
account.
(vi) Rent and insurance include:
 annual insurance premium of Rs. 800,000 for the health policy arranged by
TE for the department heads and the owner’s family members. Premium
pertaining to the owner’s family members is Rs. 200,000. The policy is valid
up to 30 June 2018.
 Rs. 1,200,000 paid against the annual rent agreement expiring on 31 August
2018. According to the rent agreement, the rent paid would not be refunded in
case the building is vacated earlier.
(vii) TE maintains provision for doubtful receivables according to the age analysis of
the outstanding balances. Relevant details are as under:
Trade receivables as on 31 December 2017 Total

Prepared by: Dawood Shahid CA & CIMA Affiliate, CPA, MPhil, MBA, OCE
Less than 3 4 - 6 7 - 12 More than
months months months 1 year
Outstanding Balances 1,970 1,000 900 530 4,400
Required Provision - 5% 10% 20%
(viii) TE depreciates property, plant & equipment at 15% per annum on reducing
balance method.
Required:
Prepare the following:
(a) Statement of profit or loss for the year ended 31 December 2017 (11)
(b) Statement of financial position as at 31 December 2017 (10)
Question-9
Following is the trial balance of Sofia Trader (ST) for the year ended 30 June 2018:
Debit Credit
---------- Rs.'000 ----------
Equipment - cost 17,000
Equipment - accumulated depreciation - 1 Jul 2017 4,800
Vehicles - cost 3,000
Vehicles - accumulated depreciation - 1 Jul 2017 1,200
Inventory - 1 Jul 2017 5,500
Trade receivables 5,350
Provision for doubtful receivables 220
Prepaid insurance - 1 Jul 2017 140
Advance 384
Bank deposit (invested on 1 Feb 2017 at 7%) 1,400
Cash and bank balances 620
Capital 16,000
Drawings 352
Trade payables 3,500
Accruals and other payables 1,520
Sales 32,350
Purchases 21,000
Returns 950 700
Salaries and utilities 2,790
Rent 1,000
Discounts 270 220
Other expenses 480
Insurance 300
Bad debts 106
Interest income on bank deposit 82

Prepared by: Dawood Shahid CA & CIMA Affiliate, CPA, MPhil, MBA, OCE
Suspense account 50
60,642 60,642
Following further information is available:
(i) Cost of closing physical inventory was Rs. 7,400,000. Inventory included
goods costing Rs. 240,000 which were damaged in the warehouse. These goods
can be sold for Rs. 250,000 after incurring a cost of Rs. 16,000.
(ii) Rent includes payment of annual rent of Rs. 240,000 expiring on 30
November 2018 for owner’s residence.
(iii) Prepaid insurance represents premium which expired on 31 January 2018
while insurance represents annual premium which is expiring on 31 January
2019.
(iv) (iv) Suspense account represents goods returned by ST which were debited
to supplier's account.
(v) On 29 June 2018, dishonoured cheque of Rs. 285,000 was returned by a
bank. No entry has been made in the books for this return. This cheque was
received from a customer net of 5% settlement discount.
(vi) ST maintains a 4% provision against trade receivables.
(vii) ST paid Rs. 388,000 (net of 3% settlement discount) which was debited to
supplier's account with the same amount.
(viii) On 1 March 2018, an equipment was sold for Rs. 400,000 and its sale
proceeds were credited to the equipment account. This equipment had been
purchased at a cost of Rs. 500,000 and on 1 July 2017 its book value was
Rs. 360,000.
(ix) Advance represents 40% payment made for purchase of vehicle. Remaining
balance would be paid in September 2018. No entry was passed when the
vehicle was delivered on 1 June 2018.
(x) ST depreciates equipment and vehicles at 15% and 25% respectively using
reducing balance method.
Required:
a) Prepare statement of profit or loss for the year ended 30 June 2018
b) Prepare statement of financial position as at 30 June 2018 (Marks 22)
Question-10
Following is the summarized trial balance of Delta Enterprises (DE), for the year
ended 30 June 2019:
Rs. in '000
Description Debit Description Credit
Property, plant and equipment 230,600 Capital 179,500
Inventory - 30 June 2019 67,800 12% Bank loan 90,000
Cash and bank balances 4,600 Accumulated depreciation 44,300
Trade receivables 94,800 Trade payables 41,400
Prepayments 3,000 Sales revenue 487,800
Other receivables 5,300 Other income 2,600
Cost of sales 354,700
Selling expenses 29,400
Administration expenses 25,900
Depreciation 19,800

Prepared by: Dawood Shahid CA & CIMA Affiliate, CPA, MPhil, MBA, OCE
Interest on bank loan 9,000
Bank charges 700
845,600 845,600
Additional information:
(i) Goods returned by a customer on 30 June 2019 were recorded on 1 July 2019.
These goods had been sold on credit for Rs. 1,950,000 at cost plus 30%.
(ii) On 1 October 2018, a printer was acquired on rent from Qazi & Co. The annual
rent of Rs. 480,000 was paid in advance and debited to prepayments. However,
the printer was purchased by DE on 1 April 2019 for Rs. 1,240,000. The
payment net of rent adjustment was made in July 2019. The purchase has not
been accounted for. DE depreciates office equipment at 20% using reducing
balance method.
(iii) DE allocates depreciation to selling and administration in the ratio of 4:6
respectively.
(iv) Annual fire insurance premium of Rs. 4,500,000 was paid in advance on 1
January 2019 and debited to administration expenses. The payment also
includes Rs. 1,350,000 pertaining to the owner's personal property.
(v) Other income includes Rs. 900,000 collected on 1 April 2019 in respect of a
service agreement for the six months ending 30 September 2019.
(vi) An amount of Rs. 960,000 receivables on 1 August 2019 on completion of a
service agreement for the six months ended 31 July 2019 has not been accounted
for.
(vii) 12% bank loan was acquired on 1 April 2018. The principal is repayable in ten
equal yearly instalments commencing from 1 April 2019. Interest is payable on 1
October and 1 April each year.
Required:
a) Prepare statement of profit or loss for the year ended 30 June 2019.
b) Prepare statement of financial position as at 30 June 2019. (Marks 18)
Question-11
Following is the summarized trial balance of Bajang Aamad Limited (BAL) for the
year ended 30 June 2020:
Head office building 15,000 1,900
Delivery vehicles 9,350 4,500
Opening stock 5,500
Trade receivables 5,255
Provision for doubtful trade receivables 400
Advance 2,160
Cash and bank balances 1,255
Capital 17,460
Long-term loans 3,000
Trade payables 2,900
Short-term loan 1,500
Sales revenue 39,770
Purchases 26,200
Operating expenses (excluding depreciation) 3,510
Sales supplies 840
Sales offices rent 1,800

Prepared by: Dawood Shahid CA & CIMA Affiliate, CPA, MPhil, MBA, OCE
Financial charges 560
71,430 71,430
Additional information:
(i) Cost of closing stock was Rs. 7,400,000, of which goods costing Rs. 500,000
were slightly damaged and can be sold for Rs. 600,000 after incurring
repacking cost of Rs. 160,000.
(ii) 60% of the sales offices rent represents payment of annual rent upto 30
September 2020.
(iii) Advance from a customer amounting to Rs. 475,000 (net of 5% discount) has
been included in sales. Goods will be delivered to the customer after year-end.
(iv) Advance represents 60% payment made on 1 February 2020 for the purchase
of a delivery vehicle. Remaining balance would be paid in September 2020. No
entry was made when the vehicle was delivered on 1 April 2020.
(v) Delivery vehicles’ cost includes an amount of Rs. 150,000 paid in June 2020
for repainting of an old delivery vehicle. However, due to heavy workload,
vehicle was not painted till year-end.
(vi) BAL depreciates building at 4% using straight line method while delivery
vehicles are depreciated at 15% using reducing balance method. 30% of cost
of the building represents land.
(vii) A specific provision at 75% is to be made against Asad Baker’s balance of Rs.
380,000. A general provision for doubtful receivables is maintained at 4% of
the remaining trade receivables.
(viii) Unused sales supplies at year-end amounted to Rs. 320,000.
(ix) The amount of operating expenses (excluding depreciation) is to be allocated
into selling and administration expenses in the ratio of 60:40 respectively.
Required:
(a) Prepare statement of profit or loss for the year ended 30 June 2020. (12)
(b) Prepare statement of financial position as at 30 June 2020. (08)
Question-12
Following is the summarized trial balance of Valsafe Traders (VT) for the year
ended 31 December 2020:
Debit Credit
-------- Rs. in '000 ------
--
Property, plant and equipment 32,500
Accumulated depreciation at 31 December 2020 - 8,750
Inventory at 1 January 2020 34,300 -
Trade receivables 36,800 -
Prepayments 780 -
Advances 240 -
Cash and bank balances 5,550 -
Capital - 60,395
Drawings 8,480 -
14% Loan - 17,500
Trade and other payables - 28,740
Revenues - 181,100
Purchases 149,800 -
Selling expenses 12,450 -

Prepared by: Dawood Shahid CA & CIMA Affiliate, CPA, MPhil, MBA, OCE
Distribution expenses 8,800 -
Administrative expenses 7,460 -
Rent income - 1,125
Interest on bank overdraft 450 -
297,610 297,610

Additional information:
(i) Cost of closing inventory in hand on 31 December 2020 amounted to Rs. 46,300,000.
Physical inventory count revealed that goods costing Rs. 950,000 were returned by a
customer for which no entry has been made. These goods were sold for Rs. 1,300,000
on credit in last week of December 2020. VT determined that these goods are out
of fashion and can be sold at 40% of original selling price.
(ii) During the year, goods having cost and selling price of Rs. 500,000 and Rs 800,000
were withdrawn by the owner for personal use but were not recorded.
(iii) Prepayments represent fire insurance premium amounting to Rs. 480,000 and Rs.
300,000 paid for office and owner's personal premises respectively. These annual
policies are valid upto 31 March 2021.
(iv) On 1 June 2020, a machine was given at a quarterly rent of Rs. 375,000, receivable in
advance.
(v) Advances represent three months’ advance salary taken by a salesman on 1
November 2020.
(vi) The loan was acquired on 1 July 2020 and the entire principal along with interest is
repayable on 30 November 2021.
(vii) Depreciation of property, plant and equipment has been incorrectly calculated at 5% on
reducing balance method instead of 10% on straight line method. The incorrect
depreciation has been included in distribution expenses though depreciation should
have been equally divided between selling and administrative expenses.
(viii) A balance of Rs. 320,000 included in trade and other payables needs to be written
back as it is no more payable.
(ix) Cash and bank balances include bank overdraft of Rs. 1,490,000.

Required:
(a) Prepare statement of profit or loss for the year ended 31 December 2020. (10)
(b) Prepare statement of financial position as at 31 December 2020. (10)

Question-12
Following is the summarized balance of onion traders (OT) for the year ended 30 June 2021
Description Debit Description Credit
Rs. in Rs. in
million million
Property plant and equipment 142.0 Accumulated depreciation as on 66.5

Prepared by: Dawood Shahid CA & CIMA Affiliate, CPA, MPhil, MBA, OCE
30 June 2021
Inventory 89.6 Provision for doubtful debts 4.5
Trade debtors 42.0 Capital 99.4
Prepayments 9.0 Long term loan 55.4
Bank deposit account 40.0 Trade debtors 33.4
Bank current account 8.3 Accruals 9.4
Owner’s drawing 17.8 Sales 397.6
Cost of sales 245.0
Operating expenses 63.7
Financial charges 8.8
666.2 666.2

Additional information
1. During the physical inventory count carried out on 30 June 2021, following matters were
noted for which nothing has been recorded in the books
 Goods having cost and selling price of Rs. 4.2 million and Rs. 5.8 million
respectively were withdrawn by the owner for his personal use.
 Goods costing Rs. 14.0 million were found damaged and would be sold at 90% of
purchase cost after incurring a rectification cost of Rs. 1.9 million.
 Goods purchased on credit costing Rs. 3.5 million were returned to a supplier on
30 June 2021
2. A payment of Rs. 7.2 million being 40% amount of a yearly maintenance contract
commenced from 1st October 2020, was included in prepayments remaining amount is
payable on completion of contract.
3. the bank deposit account was opened on 1 October 2020 for 1.5 year at 12% per
annum. interest is credited at every 6 months. Interest received was credited to sales.
4. The figure in the trial balance for the bank current account represent balance prior to the
reconciliation with bank statement. Upon comparison with bank statement, it was
discovered that:
 cheque of Rs. 3.1 million deposited into bank were not credited.
 As per standing instruction to the bank, quarterly office rent (till 31 august 2021)
amounting to Rs. 4.2 million was debited.
 Bank charges of Rs. 0.2 million were debited.
 A cheque of Rs.0.9 million was returned by bank as dishonoured. this cheque was
received from a customer net of 10% settlement discount.
5. On 30 June 2021 fully depreciated assets costing Rs. 8.2 million with a residual value of
Rs. 2.1 million were sold on credit for Rs. 2.6 million. No entry in this respect has been
recorded in the books.
6. In addition to a specific provision of Rs. 3.9 million against trade debtors of Rs. 13
million. OT needs to maintain a general provision for doubtful debtors at 5% of the
remaining closing debtor balance.
Required

Prepared by: Dawood Shahid CA & CIMA Affiliate, CPA, MPhil, MBA, OCE
a) Prepare statement of profit or loss for the year ended 30 June 2021.
b) Prepare statement of financial position as on 30 June 2021.

Prepared by: Dawood Shahid CA & CIMA Affiliate, CPA, MPhil, MBA, OCE
Solution to Question No. 1
Salman
Statement of Comprehensive Income
For the year ended 30-06-2014
Rs. Rs.
Sales (353,300 – 10,000 – 243) 340,868
Cost of Sales:
Opening Stock 127,762
Add: Purchases (330,530 – 500) 330,030
Add: Carriage Inwards 10,420
Less: Closing Stock (237,500) (230,712)
Gross Profit 110,156
Other Income
Interest Received 11,930
Other Income 3,100
Operating Expenses:
Interest 800
Power and utilities 6,500
Salaries (40,400 + 1,800) 42,200
Rent 6,992
Insurance (2,000 – 650) 1,350
Advertisement 4,512
Miscellaneous Expenses 4,130
Depreciation (4,550 + 2,120) 6,670
Bad debts 1,288 (74,442)
Net Profit 50,744
Salman
Statement of Financial Position
As on June 30, 2006
Rs. Rs. Rs.
Non-Current Assets:
Goodwill 20,000
Plant & Machinery (45,000 – 4,550 + 500) 40,950
Factory Building (42,400 – 2,120) 40,280
Current Assets:
Stock 237,500
Trade Receivable 109,420
Allowance for bad debts (2,188) 107,232
Bills Receivable 32,526
Prepaid Insurance 650
Cash at Bank 13,512
Cash in Hand 728
493,378

Prepared by: Dawood Shahid CA & CIMA Affiliate, CPA, MPhil, MBA, OCE
Capital & Liabilities:
Capital 300,000
Add: Net Profit 50,744
Less: Drawings (30,500 – 1,800) (28,700) 322,044
Non-Current Liabilities:
Loan from Bank 40,000
Current Liabilities:
Bills Payable 23,150
Accrued Expenses 3,460
Creditors 104,724
493,378
Working & Adjustments
Rs. Rs.
(i) Machine 500
Purchases 500
(ii) Salary (150 × 12) 1,800
Drawings 1,800
(iii) Prepaid Insurance 650
Insurance Expense 650

(iv) Calculation of Depreciation:


Plant & Machinery = 45,000 × 10% 4,500
Add: 500 × 10% 50
4,550
Factory Building = 42,400 × 5% 2,120
(v) Allowance
Rs. Rs.
b/d 900
Bad debts (balance) 1,288
c/d (109,420 × 5%) 2,188

Rs. Rs.
Bad debts 1,288
Allowance 1,288
Solution-2
Salman
Statement of Comprehensive Income
For the year ended 30-06-2014
Rs. Rs.

Prepared by: Dawood Shahid CA & CIMA Affiliate, CPA, MPhil, MBA, OCE
Sales (353,300 – 10,000 – 243) 340,868
Cost of Sales:
Opening Stock 127,762
Add: Purchases (330,530 – 500) 330,030
Add: Carriage Inwards 10,420
Less: Closing Stock (237,500) (230,712)
Gross Profit 110,156
Other Income
Interest Received 11,930
Other Income 3,100
Operating Expenses:
Interest 800
Power and utilities 6,500
Salaries (40,400 + 1,800) 42,200
Rent 6,992
Insurance (2,000 – 650) 1,350
Advertisement 4,512
Miscellaneous Expenses 4,130
Depreciation (4,550 + 2,120) 6,670
Bad debts 1,288 (74,442)
Net Profit 50,744
Salman
Statement of Financial Position
As on June 30, 2014
Rs. Rs. Rs.
Non-Current Assets:
Goodwill 20,000
Plant & Machinery (45,000 – 4,550 + 500) 40,950
Factory Building (42,400 – 2,120) 40,280
Current Assets:
Stock 237,500
Trade Receivable 109,420
Allowance for bad debts (2,188) 107,232
Bills Receivable 32,526
Prepaid Insurance 650
Cash at Bank 13,512
Cash in Hand 728
493,378
Capital & Liabilities:
Capital 300,000
Add: Net Profit 50,744
Less: Drawings (30,500 – 1,800) (28,700) 322,044
Non-Current Liabilities:

Prepared by: Dawood Shahid CA & CIMA Affiliate, CPA, MPhil, MBA, OCE
Loan from Bank 40,000
Current Liabilities:
Bills Payable 23,150
Accrued Expenses 3,460
Creditors 104,724
493,378
Working & Adjustments
Rs. Rs.
(i) Machine 500
Purchases 500
(ii) Salary (150 × 12) 1,800
Drawings 1,800
(iii) Prepaid Insurance 650
Insurance Expense 650

(iv) Calculation of Depreciation:


Plant & Machinery = 45,000 × 10% 4,500
Add: 500 × 10% 50
4,550
Factory Building = 42,400 × 5% 2,120
(v) Allowance
Rs. Rs.
b/d 900
Bad debts (balance) 1,288
c/d (109,420 × 5%) 2,188

Rs. Rs.
Bad debts 1,288
Allowance 1,288
Solution-3
Fortune Trader
Statement of Comprehensive Income
For the year ended 30 June 2016
Rs. Rs.
Sales (W-3) 28,761
Cost of Sales:
Opening Stock 1,045
Purchases (16,000 – 220) 15,780
Closing Stock (1,560) (15,265)

Prepared by: Dawood Shahid CA & CIMA Affiliate, CPA, MPhil, MBA, OCE
Gross Profit 13,496
Expenses
Selling & Admin Expenses (5,855-430/2 adj.(v)-240 adj.(v)+137 (6,130)
adj.(v)+89 adj.(vi)+113 (W-1)+391(W-1))
Finance Charges (W-4) (300) (6,430)
Other income
Provision for doubtful debts (W-2) 30
Net Profit 7,096
Fortune Trader
Statement of Comprehensive Income
As on 30 June 2016
Rs.000 Rs.000
Assets
Non-Current Assets
Building 37,00
Accumulated Depreciation (1,436 + 113) (1,549)
Plant & Machinery (6,650 – (W-1) 330) 6,320
Accumulated Depreciation (2,414 – 391) (2,805) 5,666

Current Assets
Trade Receivables (W-2) 3,551
Provision for doubtful debts (150)
Closing Inventor 1,560
Prepayments and other receivables 380
Cash and Bank balances 5,850
Advance for machine (W-1) 330
Fire insurance prepaid (430/2) 215 11,736
17,402

Equity & Liabilities


Capital
Opening balance 3,000
Profit for the year 7,096
Drawings (240) 9,856

Long-term Loans (5,000 – (5,000/8 x 1) 4,375


Current Liabilities
Current Maturity of long term loan (5,000 – 4,375) 625
Trade and other payables (2,080 – (W-5) 60) 2,020
Interest payable 300
Rent payable 137
Salary payable 89 3,171
17,402
WORKINGS:

Prepared by: Dawood Shahid CA & CIMA Affiliate, CPA, MPhil, MBA, OCE
(W-1)
Depreciation and rectification adjustment of advance
Building (3,700 – 1,436) x 5% 113
Plant & machinery (6,320 – 2,414) x 10% 391

Rectifying Original Wrong


Advance for machine 330 Adv. For machine 330 Plant & machinery 330
Plant & machinery 330 Cash 330 Cash 330
(W-2)
Debtor
Unadjusted 3,600 Sales return (35/100) x 140) 49
Unadjusted cl. (Bal.) 3,551
Provision
P/L (bal.) 30 b/d 180
c/d (180 – 30) 150
(W-3)
Sales 28,900
Sales return (90 + 49 (W-2)) (139)
28,761
(W-4)
Finance charges (5,000 x 12% x 6/12) 300
(W-5)
Rectifying Original Wrong
Trade payables 60 Trade payable 30 Trade payable 30
Suspense 60 Purchase return 30 Purchase return 30
Solution-4
Azam Autoparts Store
Statement of Comprehensive Income
For the year ended 30 June 2015
Rs.000 Rs.000
Sales 131,943
Cost of Sales:
Opening Stock 14,500
Purchases (105,950 – 4,000) 101,950
Closing Stock (24,090) (92,360)
Gross Profit 39,583
Salaries expenses 10,250

Prepared by: Dawood Shahid CA & CIMA Affiliate, CPA, MPhil, MBA, OCE
Depreciation expenses (W-1) 3,455
Operating expenses (W-2) 7,029
Interest expenses 600
1,060
Utility expenses
1,205
Vehicle expenses
(23,599)
Net Profit
15,984
Azam Autoparts Store
Statement of Comprehensive Income
As on 30 June 2015
Non-Current Assets
Fixed Assets (W-1) 13,820
Current Assets:
Stock in trade 24,090
Prepaid insurance 302
Prepaid rent 500
Cash and bank 8,550
47,262
Liabilities and Equity
Opening capital 27,883
Add: Net Profit 15,984
Less: Drawings (W-3) (4,215)
39,652
Current Liabilities
Loan from Bank 5,000
Creditors 2,500
Interest payable 50
Accrued expenses 60
47,262

W-1: fixed assets and depreciation Rs.000


Shop equipment purchased 1,400
Fixture purchased 5,500
6,900
Depreciation (6,900 @ 20%) 1,380
5,520
Shop equipment balance (5,880 – 1,400) 4,480
Motor vehicles 3,820
13,820
Depreciation as per trial 2,075
Add: depreciation shop equipment and fixtures 1,380
3,455
Rs. 000
W-2: Operating expenses

Prepared by: Dawood Shahid CA & CIMA Affiliate, CPA, MPhil, MBA, OCE
As per trial 6,046
Add: prepaid rent to be recorded as expense 1,500
Less: prepaid insurance (302)
Less: personal expenses (215)
7,029

W-3: Drawings Rs. 000


Drawings of goods by Azam 4,000
Insurance 215
4,215
Solution-5
Falcon Traders
Trading and Profit and Loss Account
For the year ended 31 December 2014
Sales ((W-1) 1,125,000 + 375,000) 1,500,000
Less: Cost of sales (W-6) (1,116,000)
Gross Profit 384,000
Less: Admin Expenses
Salaries 38,000
Selling expense 45,000
Office expense (W-4) 23,000
Disc. Allowed 21,000
Interest Expense (190,000 x 10%) 19,000
Provision for Doubtful Debts (W-5) 12,300
Depreciation
Building 61,000
Plant and Machinery (W-10) 51,570
Office Equipment (W-16) 19,000
Loss on disposal 15,000
(304,870)
Other income (Discount received) 13,000
Net Profit 92,130
Falcon Traders
Balance Sheet
As on 31 December 2014
Capital and Liabilities
Capital
Opening Capital (W-9) 1,260,000
Capital Introduces -
Add: Net Profit 92,130
Less: Drawings
1,352,130
Bank Loan(190,000 – 25,000) 165,000
Current Liabilities
Trade Creditors (W-2) 85,500

Prepared by: Dawood Shahid CA & CIMA Affiliate, CPA, MPhil, MBA, OCE
Payable for expense 28,000
113,500
Total 1,630,630

Assets
Non-Current Assets
Freehold Land 100,000
Building 549,000
Plant and Machinery 392,540
Office Equipment 76,000
1,117,540
Current Assets
Debtors 233,700
Stocks 90,000
Bank 172,440
Cash 16,950
513,090
Total 1,630,630
(W-1)
Debtor Account
Opening 177,000 Bank 1,035,000
Sales 1,125,000 Discount allowed 21,000
(375,000/25% x 75%) Cl. (bal.) 246,000
(W-2)
Creditor Account
Bank 641,450 Opening 75,000
Discount Received 13,000 Purchases 664,950
Closing balance 85,500
(W-3)
Bank Account
Opening 45,000 Creditors 641,450
Disposal 35,690 Salaries 38,000
Debtors 1,035,000 Selling expense 45,000
Disposal (W-8) 20,000 Office expense 30,000
Interest expense 19,000
P&M A/C 164,800
Loan (repaid) 25,000

Prepared by: Dawood Shahid CA & CIMA Affiliate, CPA, MPhil, MBA, OCE
Closing (bal.) 172,440
(W-4)
Office Expense
Bank 30,000 Opening 35,000
Closing 28,000 P/L (Bal.) 23,000
(W-5)
Provision for Doubtful Debts
Opening
Closing (246,000 x 5%) 12,300 P/L(Bal.) 12,300

(W-6)
Inventory account
b/d 183,000 COS (bal.) 1,116,000
Creditor 664,950
Cash 358,050 Closing inventory 90,000
(664,950/65% x 35%)

(W-7)
Plant and Machinery account-BV
b/d 330,000 Disposal A/c 50,690
Depreciation (W-10) 51,570
Bank 164,800 Closing (bal.) 392,540
(W-8)
Disposal account
Machinery-BV 50,690 Bank (bal.) 35,690
P/L 15,000
(W-9)
Opening Assets (100,000+610,000+330,000+115,000+183,000+177,000+45,000)
1,560,000
Opening Liabilities (190,000+75,000+35,000) (300,000)
Opening Capital 1,260,000
(W-10)
Depreciation of Plant & Machinery:
On opening excluding Disposal (330,000-54,800) x 15% 41,280
On additions (164,800 x 15% x 3/12) 6,180
On disposals (54,800 x 15% x 6/12) 4,110
51,570
(W-11)

Prepared by: Dawood Shahid CA & CIMA Affiliate, CPA, MPhil, MBA, OCE
Disposal WDV
54,800 – (W-10) 6,180) 48,620
(W-12)
Particulars Dr. Cr.
Interest exp. 19,000
Loan 25,000
Bank 44,000
(W-13)
Cash
Sales 375,000 Inventory 358,050
Cl. (bal.) 16,950
(W-14)
Office Equipment-BV
b/d 115,000 Disposal 20,000
Depreciation (W-16) 19,000
Cl. (bal.) 76,000
(W-15)
Disposal account
Office equipment-BV 20,000 Cash 20,000
P/L (bal.) -
(W-16)
Depreciation of office equipment
(115,000 – 20,000) x 20% 19,000
Solution-6
Moon Trading
Statement of Comprehensive Income
For the year ended 31 December 2015
Sales Revenue (2,840 - 2.4) 2,837.60
Less: Cost of sales
Opening inventory 410
Purchases 2,135.40
Closing inventory (529.22)
(2,019)
Gross Profit 821.42
Less: Selling and Admin Expenses (W-1) (539.92)
Less: Finance Charges (8 + (160x 10% x 3/12) + 2 (14.25)
+0.25) 23.75
Other income 291.00
Net Profit before tax

Moon Trading

Prepared by: Dawood Shahid CA & CIMA Affiliate, CPA, MPhil, MBA, OCE
Statement of Financial Position
As on 31 December 2015
Equity and Current Liabilities
Capital:
Opening Capital (W-9) 50
Profit for the year 291
Less: Drawings (0.32)
340.68
Long term Loan 160
Current Liabilities
Trade Creditors (320 – 4.5) 315.50
Accruals and other payables 25 +(160x 10% x3 + 12)+(1.8 29.20
(3.6x 4/9)
Total 845.38
Assets
Non-Current Assets
Fixed assets (150 + 0.78) 150.78
Accumulated Depreciation (45 + 0.04) (45.04)
105.74
Current Assets
Trade Debtors (200 + 3.5 – 2.4) 201.10
Provision for doubtful debts (201.1 x 3%) (6.03)
Closing inventory 529.22
Prepayments, advances and other receivable 10.35
9+1.2+0.45+0.6
Cash and bank balances 5.00
Total 845.38
(W-1) Selling and administration expenses:
Rs. in
“Million”
As per trial 540
Add: Dep. on items in office use 0.04
Add: Increase in Provision 0.03
Less: Office supplies in hand (0.15)
539.92
(W-2)
Provision for doubtful debts
Rs. In Rs. In
million million
P/L a/c (bal.) 0.03 b/d 6
c/d (200-2.4+3.5) 3% 6.03
Solution-7
Rainbow Lights
Statement of Comprehensive Income
For the year ended 31 December 2016

Prepared by: Dawood Shahid CA & CIMA Affiliate, CPA, MPhil, MBA, OCE
Sales Revenue [750–(20×5%)–7] 742
Less: Cost of sales [304+2–(7/1.4)] (301)
Gross Profit 441
Less: Selling and Admin Expenses (W-1) (140.42)
Less: Finance Charges [10+(120×10%×5/12)] (15)
Other income [45–(12/12×4)] 41
Net Profit before tax 326.58
Rainbow Lights
Statement of Financial Position
As on 31 December 2016
Rs. In
Equity and Liabilities million
Equity:
Opening Capital 100
Profit for the year (326.58–30) 296.58
Less: Drawings(50+1) (51)
345.58
Long term Loan 120
Current Liabilities
Trade Creditors (240 – 8) 232
Accruals and other payables 37
[28+(120×10%×5/12)+(12×4/12)]
Total 734.58

Assets
Non-Current Assets
Property, plant and equipment - cost (270–8) 262
(149.60)
Accumulated Depreciation (150–0.4) (W.1)
112.40
Current Assets
Trade Debtors (W-2) 394
Provision for doubtful debts (W-2) (11.82)
Closing inventory [170–2–15–1+(7/1.4)] 157
Prepayments, advances and other receivable 63
[45+(24/12×9)]
Cash and bank balances 20
Total 734.58
(W-1) Selling and administration expenses
Rs. in
“Million”
As per trial 146.00
Decrease in bad debts expense W.2 (2.18)
Inventory damaged in fire (abnormal loss) 15.00
Building rent paid in advance (24/12×9) (18.00)
Reversal of depreciation on equipment returned (8×15%×4/12) (0.40)

Prepared by: Dawood Shahid CA & CIMA Affiliate, CPA, MPhil, MBA, OCE
140.42
(W-2)
Trade Provision for
debtors doubtful
debts
As per trial balance 400.00 12.00
Sales return - (7.00)
Sales net of 5% trade discount recorded at gross (1.00)
(20×5%)
Correction of recovery of previously written off debtors 2.00 2.00
394.00 14.00
Decrease in provision Balancing (2.18)
Provision at 3% of the year-end balance (394×3%) 11.82
Solution- 8
Home Appliances
Statement of Profit and Loss
For the year ended 30 June 2017
Rs in
million
Sales 101,120
Sales return (3,000+400) 3,400
Net Sales 97,720
Cost of Sales:
Opening Inventory 8,200
Purchases 70,000
Carriage Inwards 1,360
Available for sale 79,560
Closing Inventory (9,310)
(70,250)
Gross Profit 27,470
Operating expenses:
Wages and salaries 5,900
Discounts 1,600+50 [(940+10)÷95%×5%] 1,650
Utility charges 2,350
Running and maintenance charges (3,290+110) 3,400
Financial Charges (580+10) 590
Depreciation expense: Excl. sold machine [34,400 − (8,000 60)]×20% 5,292
Sold machine (600-60)×20%×11÷12
*99
(19,281)
Other income:
Machine rent income 120×8.5÷12 85
Gain on disposal of machine (600×20%×6÷12) 450-WDV i.e. [600–(99+60)] 9

Prepared by: Dawood Shahid CA & CIMA Affiliate, CPA, MPhil, MBA, OCE
Decrease in provision for doubtful debts 750−555 195
Misc. income 110
399
Net profit 8,588

Home Appliances
Statement of Financial Position
As on 30 June 2017
Equity and Current Liabilities
Capital:
Opening Capital 25,500
Profit for the year 8,588
Less: Drawings (4,500)
29,588

Current Liabilities
Trade Creditors 10,200
Accruals and other payables (5,310 – 120+110) 5,300
Total 45,088

Assets
Non-Current Assets
Fixed assets 35,000 – (540÷90%) 34,400
Accumulated Depreciation 8,000 + [(5,292+99) – (60+99) (13,232)
21,168
Current Assets
Trade Debtors [12,500–400] – [(940+10)÷95%] 11,100
Provision for doubtful debts (11,100×5%) (555)
Closing inventory 9,310
Prepayments and other receivable 2,350+[450 – (120×3.5÷12)] 2,765
Cash and bank balances 1,300
Total 45,088
Solution- 9
Tulip Enterprises
Statement of Profit and Loss
For the year ended 31 December 2017
Sales 35,230
Cost of sales 23,580+70{1,000-(1,130-200)} (23,650)
Gross profit 11,580
Selling and administration expenses:
Salaries and wages (2,610)
Fuel and power (450)
Bad debt expense 230+[ 246(BS)-220] (256)

Prepared by: Dawood Shahid CA & CIMA Affiliate, CPA, MPhil, MBA, OCE
Rent and insurance 2,900-200-300{(800-200)×0.5)}-800(1,200×8÷12) (1,600)
Depreciation expense [1,180.5{(12,500-4,630)×0.15}]+{16.88(450×0.15×3÷12)} (1,197)
Repair and maintenance (920)
(7,033)
Miscellaneous income 940-120(480×0.5×3÷6)+150 970
Profit from operation 5,517
Financial charges (700)
Net profit 4,817
Tulip Enterprises
Statement of Financial Position
As on 31 December 2017
Assets
Non-current assets
Property, plant and equipment – cost 12,500+450 12,950
Accumulated depreciation 4,630+ 1,197(PL) (5,827)
7,123
Current assets
Trade receivables 4,400
Provision for doubtful debts (1,000×0.05)+(900×0.1)+(530×0.2) (246)
4,154
Stock-in-trade 3,900-70(PL)-670 3,160
Prepayments & advances 1,740+300(PL)+800(PL)-450 2,390
Other receivables 150
Cash and bank 2,320
12,174
19,297
Equity and liabilities
Equity
Capital 6,000
Net profit 4,817
Drawings 490 +200 (690)
10,127
12% Long-term loan 5,150-150(5,150×3÷103) 5,000
Current liabilities
Trade payables 3,250-670 2,580
Accruals and other payables 1,320+120(PL)+150(BS) 1,590
4,170
19,297
Solution- 10

Prepared by: Dawood Shahid CA & CIMA Affiliate, CPA, MPhil, MBA, OCE
Sofia Traders
(a)Statement of profit or loss for the year ended 30 June 2018
Rs. in '000'
Revenues
Sales 32,350
Return inward (950)
Net sales 31,400

Cost of sales
Opening inventory 5,500
Purchases 21,000
Return outward 700+50 (750)
Closing inventory 7,400–6{240–(250–16)} (7,394)
Cost of sales (18,356)
Gross profit 13,044

Selling and administration expenses:


Salaries and utilities 2,790
Rent 1,000–240 760
Discount allowed 270–15[300(BS)–285] 255
Insurance 300–175(300×7/12)+140 265
Bad debts 106+{226(BS)–220} 112
Depreciation - equipment 1,872
Disposal(500–140)×15%×8/12 = 36
Others[(17,000+400–500)–(4,800–140)]×15% = 1,836
Depreciation- vehicle 470
Addition960(BS)×25%×1/12 = 20
Others(3,000–1,200)×25% = 450
Other expenses 480
(7,004)

Other income
Gain on disposal 400–[360–36(PL)] 76
Discount received 220+12(388/97×3) 232
Interest income 82+16{1,400×7%–82} 98
406
Net profit 6,446

Statement of financial position as at 30 June 2018 Rs. in '000


Assets
Non-current assets
Equipment - cost 17,000+400–500 16,900
Accumulated depreciation 4,800+1,872(PL)–176(140+36) (6,496)
10,404

Vehicle - cost 3,000+960(384/0.4) 3,960


Accumulated depreciation 1,200+470(PL) (1,670)
2,290

Prepared by: Dawood Shahid CA & CIMA Affiliate, CPA, MPhil, MBA, OCE
Current assets
Trade debtors 5,350+300(285/0.95) 5,650
Provision of doubtful debts (5,650×4%) (226)
5,424
Closing inventory 7,400–6(PL) 7,394
Prepayments PL 175
Accrued income/Interest receivable PL 16
Deposits at 7% 1,400
Cash and bank balances 620–285 335
13,344
27,438

Equity and liabilities


Equity
Capital 16,000
Profit for the year 6,446
Drawings 352+240 (592)
21,854

Current liabilities
Trade creditors 3,500–12(PL) 3,488
Accruals and other payables 1,520+576[960(BS)×0.6] 2,096
5,584
27,438
Solution- 11
(a) Delta Enterprises
Statement of profit or loss for the year ended 30 June 2019
Rs. in '000'
Net sales 487,800-1,950 485,850
Cost of sales 354,700–1,500(1950÷1.3) (353,200)
Gross profit 132,650
Selling W-1 (37,345)
expenses
Administration expenses W-1 (35,132)
Financial 9,000+2,700(90,000×0.12×3÷12)+700 (12,400)
charges
Other income 2,600–450(900×3÷6)+800(960×5÷6) 2,950
Net profit 50,723

(b) Statement of financial position as at 30 June 2019


Rs. in '000'
Assets
Non-current assets
Property, plant and equipment (230,600+1,240)–(44,300+62(W-1)) 187,478

Current assets
Inventory 67,800+1,500(PL) 69,300

Prepared by: Dawood Shahid CA & CIMA Affiliate, CPA, MPhil, MBA, OCE
Trade receivables 94,800–1,950 92,850
Prepayments 3,000+1,575(W-1) –240–240 4,095
Other receivables 5,300+800(PL) 6,100
Cash and bank balances 4,600
176,945
364,423
Equity and liabilities
Equity
Capital 145,000
Profit or loss 34,500+PL 85,223
Drawings (1,350)
228,873

Long-term loan 90000×8÷9 80,000

Current liabilities
Trade & other payables 41,400+(1,240–480×6÷12) 42,400
Unearned revenue PL 450
Interest payable PL 2,700
Current maturity of long- 90,000×1÷9 10,000
term loan
55,550
364,423

W-1: Selling and administration expenses Selling Administration


-------- Rs. in ‘000 --------
As given 29,400 25,900
Depreciation 7,945 11,917
[19,800+62(1,240×0.2×3÷12)] in 40:60
Printer's rent 480×6÷12 - 240
Insurance pertaining to the owner's property (1,350)
Prepaid [(4,500–1,350)÷2] - (1,575)
insurance
37,345 35,132
Solution- 12
(a) Bajang Aamad Limited
Statement of profit or loss for the year ended 30 June 2020
Rs. in '000
Revenues 39,770–475 39,295
Cost of sales (W-1) (24,360)
Gross profit 14,935
Selling expense (W-2) (4,996)
Administration expense (W-2) (1,904)
Operating profit 8,035
Finance cost (560)
Net profit 7,475

W-1: Cost of sales Rs. in '000

Prepared by: Dawood Shahid CA & CIMA Affiliate, CPA, MPhil, MBA, OCE
Opening stock 5,500
Purchases 26,200
Closing inventory 7,400–60[500–(600–160)] (7,340)
24,360

W-2: Selling & administrative expense Selling Administrati


ve
-------- Rs. in '000 --------
Operating expenses 3,510 in 60:40 2,106 1,404
Sales office rent 1,800–270(1,800×0.6×3÷12) 1,530
Depreciation (W-3) 840 420
Bad debt (480(W-5)–400) 80
Sales supplies 840-320 520
4,996 1,904

W-3: Depreciation expense Rs. in '000


Vehicle:
On opening balance (9,350–150–4,500)×15% 705
On addition 3,600×15%×3/12 135
840
Building 15,000×0.7×4% 420
1,260

(b) Statement of financial position as at 30 June 2020


Rs. in '000
Assets
Non-current assets
Property, plant and (W-4) 20,140
equipment

Current assets
Stock in trade (W-1) 7,340
Sales supplies 320
Trade receivables – net (W-5) 4,775
Prepayment 270 PL +150 PL 420
Cash and bank balances 1,255
14,110
34,250
Equity and liabilities
Equity
Share capital (Rs. 10 each) 8,000
Share premium 2,800
Retained earnings 6,660+7,475 14,135
24,935
Non-current liabilities
Long-term loan 3,000

Prepared by: Dawood Shahid CA & CIMA Affiliate, CPA, MPhil, MBA, OCE
Current liabilities
Trade and other payables 2,900+1,440(3,600 –2,160) 4,340
Advance from customer 475
Short term loan 1,500
6,315
34,250

W-4: Property, plant & equipment Rs. in '000


Head office building 15,000
Accumulated depreciation 1,900+420(W-3) (2,320)
12,680
Delivery vehicles 9,350+3,600(2160÷0.6)–150 12,800
Accumulated depreciation 4,500+840(W-3) (5,340)
7,460
20,140

W-5: Trade receivables – net Rs. in '000


Trade receivables 5,255
Provision for doubtful 380×75%+( 5,255–380)×4% (480)
receivables
4,775
Solution- 13
Valsafe Traders
Statement of profit or loss for the year ended 31 December 2020
Rs. in '000
Revenue 181,100–1,300 179,800
Cost of goods sold
Opening inventory 34,300
Purchases 149,800
Closing inventory 46,300–950+520(40%×1,300) (45,870)
Goods withdrawn (500)
Cost of goods sold (137,730)
Gross profit 42,070
Selling expense 12,450+160(240×2÷3)+1,625(32,500×10%×50%) (14,235)
Distribution expense 8,800–1,250{(32,500–8,750)×5%÷95%} (7,550)
Administration expense 7,460+360(480×9÷12)+1,625 (9,445)
(31,230)
Operating profit 10,840
Other income 1,125–250(375×2÷3)+320 1,195
Financial charges 450+1,225(17,500×14%×6÷12) (1,675)
Net profit 10,360

(b) Statement of financial position as at 31 December 2020


Rs. in '000
Assets
Non-current assets

Prepared by: Dawood Shahid CA & CIMA Affiliate, CPA, MPhil, MBA, OCE
Property, plant and equipment 32,500
Accumulated depreciation (8,750–1,250(PL)+3,250(PL) (10,750)
21,750
Current assets
Inventory (PL) 45,870
Trade receivables 36,800–1,300 35,500
Advance 240–160(PL) 80
Prepayments 780–360(PL)–300 120
Cash and bank balances 5,550+1490 7,040
88,610
110,360
Equity and liabilities
Equity
Capital 60,395
Net profit 10,360
Drawings 8,480+500+300 (9,280)
61,475

Current liabilities
Trade and other payables 28,740–320 28,420
Short term loan 17,500
Unearned rent PL 250
Interest payable PL 1,225
Bank overdraft 1,490
48,885
110,360

Prepared by: Dawood Shahid CA & CIMA Affiliate, CPA, MPhil, MBA, OCE

You might also like